Sie sind auf Seite 1von 44

G.

S 3 KSMC 2010
1. Which of the following statements is false concerning the use of local anesthetic
agents:

A=Complications due to excessive plasma concentration can result from inadvertent


intravascular injection of the agent
B=Bupivacaine is noted for a slow onset but long duration.
C=The addition of epinephrine to a local anesthetic agent will both lower the toxicity and
increase the duration of local anesthesia
D=Hypotension observed when a local anesthetic is administered in the form of a spinal
epidural block, is the result of myocardial depression

Answer: d
Local anesthetics constitute a class of drugs which produce temporary blockage of nerve
conduction by binding to neuronal sodium channels. Adverse consequences associated
with the use of local anesthetics fall into three categories:
1- acute central nervous system toxicity due to excessive plasma
concentration,
2- hemodynamic and respiratory consequences due to excessive conduction
block of the sympathetic or motor nerves,
3- and allergic reactions.
Whenever a local anesthetic has been injected, there may be inadvertent intravascular
injection or an overdose of the drug due to rapid uptake from the tissues. All may
produce seizures. Complications can be minimized by aspirating prior to injection to
avoid intravascular injection and limiting the doses to the safe range. When local
anesthetics are administered for a spinal or epidural block, there will be a progressive
blockade of the sympathetic nervous system which will produce systemic vasodilatation.
If the block travels along the thoracolumbar region, a sympathetic blockade will result
in profound systemic vasodilatation and bradycardia with resultant hypotension.
Local anesthetics are divided into two groups:
1- esters and
2- amides.
Most commonly used agents, the amides, include lidocaine and bupivacaine.
Lidocaine is noted for a fast onset of action but a short duration whereas bupivacaine
has a slower onset with the duration lasting for four to 12 hours. The addition of
epinephrine (100 µg) will lower the toxicity and increase the duration of the local
anesthetic.
----------------------------------------------------------------------------------------------------------------
-
2.Which of the following statements is true concerning the vascular response to injury?
A=Vasoconstriction is an early event in the response to injury
B=Vasodilatation is a detrimental response to injury with normal body processes working
to avoid this process
C=Vascular permeability is maintained to prevent further cellular injury
D=Histamine, prostaglandin E2 (PGE2) and prostacyclin (PGI2) are important
mediators of local vasoconstriction.

Answer: A
After wounding, there is transient vasoconstriction mediated by catecholamines,
thromboxane, and prostaglandin F2 (PGF2a). This period of vasoconstriction lasts for
only five to ten minutes. Once a clot has been formed and active bleeding has stopped,
vasodilatation occurs in an around the wound. Vasodilatation increases local blood flow
to the wounded area, supplying the cells and substrate necessary for further wound
repair. The vascular endothelial cells also deform, increasing vascular permeability. The
vasodilatation and increased endothelial permeability is mediated by histamine, PGE2,
and prostacyclin as well as growth factor VEGF (vascular endothelial cell growth factor).
These vasodilatory substances are released by injured endothelial cells and mast cells
and enhance the egress of cells and substrate into the wound and tissue.

3.Which of the following statements concerning laboratory studies used in monitoring a


patient with intravenous heparinization is correct?
A=The platelet count should be followed.
B=The prothrombin time should be observed if prolonged treatment is necessary.
C=The activated partial thromboplastin time (aPTT) should be maintained at
approximately 60-80 times normal.
D=The serum creatinine should be measured daily to allow adjustments in dose based on
renal function.

Answer: A
In monitoring the effect of heparin, an activated partial thromboplastin time (aPTT) of 1.5
control or a thrombin clotting time (TCT) of 2 times control reflects adequate
anticoagulation. The prothrombin time remains normal. Heparin-associated
thrombocytopenia from an immune mechanism is a potential complication of the use of
this anticoagulant. Therefore any patient undergoing heparin therapy should have a
platelet count determined every other day after the fourth day of therapy or earlier if he or
she is known to have been exposed to heparin in the past. Heparin is not excreted through
the kidneys or the liver but is cleared through the reticuloendothelial system. Therefore the
dose of heparin need not be adjusted in cases of liver or renal dysfunction.
----------------------------------------------------------------------------------------------------------------
-
4.Which of the following statements is true concerning heparin-associated
thrombocytopenia?
A=Heparin-associated thrombocytopenia occurs only in the face of over anticoagulation
with heparin
B=Severe thrombocytopenia (platelet count less than 100,000) is seen in 60% of patients
treated with heparin
C=Heparin-associated thrombocytopenia is due to the aggregation of platelets and may
result in thrombosis or embolic episodes.
D=Heparin-associated thrombocytopenia seen within hours of initiation of heparin
therapy.

Answer:C
Heparin-associated thrombocytopenia occurs in 0.6% to 30% of patients who receive
heparin, although severe thrombocytopenia (platelet counts less than 100,000) is seen in
fewer than 10% of patients treated with heparin. It is caused by a plasma factor, most likely
a heparin-dependent platelet antibody, that causes aggregation of platelets when exposed to
heparin. Activation of platelets in this setting results in thrombocytopenia, thrombosis and
embolic episodes, which can lead to death. Both bovine and porcine heparin have been
associated with this syndrome, which usually begins 5 to 15 days after initiating heparin
therapy. Even trivial exposure with heparin such as coating on pulmonary artery catheters
or low rate infusion into arterial catheters may cause this syndrome.
----------------------------------------------------------------------------------------------------------------
-
5.Antithrombin III deficiency is a commonly observed hypercoaguable state. Which of the
following statements is false concerning this condition?
A= patient with this deficiency usually presents with thrombosis while on coumadin or
exhibits an inability to become adequately anticoagulated with Coumadin.
B=This deficiency may be either congenital or acquired.
C=Thrombotic episodes are related to predisposing events such as operations, childbirth,
and infections.
D=Treatment involves acutely the administration of fresh frozen plasma followed by
long-term treatment with Coumadin.

Answer: A
Antithrombin III deficiency accounts for about 2% of venous thrombotic event. This
deficiency has been described in patients with pulmonary embolism, mesenteric venous
thrombosis, lower extremity venous thrombosis, arterial thrombosis, and dialysis fistula
failure. Antithrombin III is a serine protease inhibitor of thrombin and factors Xa, IXa and
XIa. Because one of the main actions of heparin is to potentiate the anticoagulant effects
of antithrombin III, a patient with this deficiency usually presents with thrombosis while
on heparin or exhibits the inability to become adequately anticoagulated with heparin. This
deficiency may be either congenital (1n2000–5000 births) or acquired. Acquired defects
occur with inadequate production, as in liver disease, malignancy, nephrotic syndrome,
disseminated intervascular coagulation, malnutrition, or increased protein catabolism.
Thrombotic episodes are related to predisposing events such as operations, childbirth, and
infections. Once the diagnosis of antithrombin III deficiency is established, fresh frozen
plasma should be administered followed by long-term treatment with Coumadin.
----------------------------------------------------------------------------------------------------------------
-
6.Mini-dose heparin has been shown to be useful in the prophylaxis of postoperative
venous thrombosis. Mechanisms by which low-dose heparin is thought to protect against
venous thrombosis include all except:
A. Enhancement of antithrombin III activity.
B. A decrease in thrombin availability.
C. Inhibition of platelet aggregation and subsequent platelet release action.
D. A prolongation of activated partial thromboplastin time.
Answer: D
Low-dose heparin is thought to protect against venous thrombosis through three different
mechanisms. First, antithrombin III activity with its inhibition of activated Factor X is
enhanced by only trace amounts of heparin; second, there is a decrease in thrombin
availability that prevents its activation and thus its fibrin-stabilizing effect; and third, small
doses of heparin may inhibit the second wave of platelet aggregation and subsequent
platelet release reaction. The standard doses of heparin administered (5000 units bid) does
not affect aPTT.
-----------------------------------------------------------------------------------------------------------
7.Tests of coagulation are used to monitor anticoagulation treatment and detect intrinsic
abnormalities in coagulation. Which of the following statements is false concerning
coagulation tests?
A=Prothrombin time (PT) measures both the intrinsic and extrinsic clotting pathways
and fibrinogen.

B=Activated partial thromboplastin time (aPTT) can be used to monitor both oral
anticoagulation with Warfarin and intravenous anticoagulation with heparin.
C=Thrombin clotting time (TCT) is a measurement of the time it takes for exogenously
administered thrombin to turn plasma fibrinogen into fibrin clot.

D=Whole blood activated clotting time (ACT) is a measurement of the ability of whole
blood to clot and is used to monitor heparin levels intraoperatively during cardiovascular
and peripheral vascular operations
Answer: b
Coagulation tests include prothrombin time (PT), which measures the intrinsic and
extrinsic pathways of fibrinogen production and is the most common method for
measuring a level of oral anticoagulant therapy. The activated partial thromboplastin
time (aPTT) identifies the abnormalities of the contact and intrinsic phases of
coagulation. Values of aPTT have variably been shown to correlate with heparin dosages
and serum heparin levels and are therefore most commonly used in monitoring heparin
therapy. It is of no value in long-term management of patients on oral Warfarin therapy.
Thrombin clotting time (TCT) is the measure of the time it takes for exogenously
administered thrombin to turn plasma fibrinogen into fibrin clot. It is extremely sensitive
to levels of heparin and is an excellent measure of measuring the level of heparin-
induced anticoagulation. The beauty of the TCT is that it is not specific for any disease
condition; thus it may be used to differentiate factor deficiencies from the presence of
heparin, or to separate lupus anticoagulant from abnormalities in fibrinogen levels. The
whole blood activated clotting time (ACT) is a measurement of the ability of whole blood
to clot, and as such, is an available technique for monitoring heparin levels
intraoperatively. The ACT responds in a linear fashion to increasing heparin dosage and
correlates well with the observed clinical anticoagulation. Adequate anticoagulation for
extracorporeal circulation is defined as an ACT of 480 seconds or more while for
peripheral vascular applications, values of 250 seconds or greater are considered
appropriate.

8.Which of the following statements is true concerning hemophilia A?


A=Hemophilia A is inherited as a sex-linked recessive deficiency of factor IX.
B=A positive family history for bleeding disorders present in all patients.
C=Laboratory tests reveal a prolongation of aPTT, prothrombin time (PT), thrombin
clotting time and platelet aggregation.
D=Spontaneous bleeding is unusual with factor VIII levels greater than 10% of normal
Answer: D
Hemophilia A is inherited as a sex-linked recessive deficiency of factor VIII although 0%
of cases are secondary to spontaneous mutation. The incidence of this abnormality is
approximately 1/10,000 births. Laboratory screening tests usually reveal a prolongation
of an aPTT but normal prothrombin time (PT), thrombin clotting time (TCT) and platelet
aggregation testing. The minimum level of VIII required for hemostasis is 30% for minor
bleeding, whereas spontaneous bleeding is unusual with factor levels greater than 5 to
10% of normal. In severe genetic deficiency states however, factor levels as low as 1%
have been noted and patients are at risk for spontaneous bleeding.
9.Von Willebrand’s disease is a common, congenital bleeding disorder. Which of the
following statements is false concerning Von Willebrand’s disease?
A=As in hemophilia, it is much more common in men.
B=A history of spontaneous bleeding is not as common as in hemophilia A.
C=Screening laboratory tests will include a prolonged aPTT with a normal prothrombin
time.
D=Pre-treatment for elective surgery require administration of cryoprecipitate to achieve
levels of 23–50% of normal

Answer: a
Von Willebrand’s factor is an adhesive protein that mediates platelet adhesion to collagen.
In addition, it protects and prevents the rapid removal of factor VIII from blood. The
classical deficiency state, Von Willebrand’s disease, is caused by reduction of factor VIII
activity (although not as great as Hemophilia A) and the Von Willebrand factor. Clinical
manifestations include epistaxis, gingival bleeding, menorrhagia, rare joint or muscle
bleeding, and subcutaneous bleeding. Spontaneous bleeding is not as common as in classic
Hemophilia A. The syndrome is transmitted as both autosomal dominant (heterozygous)
and autosomal recessive disease (homozygous) traits. Therefore there is no sex
predilection. Screening laboratory tests include a prolonged aPTT with a normal
prothrombin time. In addition, because of the importance of this factor in platelet
adhesion, patients display a prolonged bleeding time and have decreased level of factor
VIII activity, decreased immunoreactive levels of Von Willebrand’s antigen, and abnormal
platelet aggregation responses to ristocetin. The most reliable source of Von Willebrand’s
factor is cryoprecipitate
-----------------------------------------------------------------------------------------
10.Scar formation is part of the normal healing process following injury. Which of the
following tissues has the ability to heal without scar formation?
A=Liver
B=Skin
C=Bone
D=Muscle
Answer: C
Every tissue in the body undergoes reparative processes after injury. Bone has the unique
ability to heal without scar and liver has the potential to regenerate parenchyma, the only
organ that has maintained that ability in the adult human. Although liver does regenerate,
it often heals with scar (cirrhosis) as well. With these exceptions, all other mature human
tissues heal with scar.
----------------------------------------------------------------------------------------------------------------
-
11.Which of the following factors has been demonstrated to promote wound healing in
normal individuals?
A=Vitamin A supplementation.
B=Vitamin C supplementation.
C=Vitamin E application to the wound.
D=Zinc supplementation.
E=None of the above.
F=All of the above.
Answer: E
Several important systemic factors or conditions influence wound healing. Interestingly,
there are no known systemic conditions that lead to enhanced or more rapid wound
healing. Overall nutrition as well as adequate vitamins play an important role in wound
healing. Vitamin A is involved in the stimulation of fibroplasia, collagen cross-linking, and
epithelialization. Although there is no conclusive evidence in humans, vitamin A may be
useful clinically for steroid-dependent patients who have problematic wounds or who are
undergoing extensive surgical procedures. Vitamin C is a necessary cofactor in
hydroxylization of lysine and proline in collagen synthesis and cross-linkage. The utility of
vitamin C supplementation in patients who otherwise take in a normal diet has not been
established. Vitamin E is applied to wounds and incisions empirically by many patients.
The evidence to support this practice is entirely anecdotal. In fact, large doses of vitamin E
have been found to inhibit wound healing. Zinc and copper are also important cofactors
for many enzyme systems that are important to wound healing. Deficiency states are seen
with parenteral nutrition but are rare and readily recognized and treated with supplements.
Overall, vitamin and mineral deficiency states are extremely rare in the absence of
parenteral nutrition or other extreme dietary restrictions. There is no evidence to support
the concept that supranormal provision of these factors enhance wound healing in normal
patients.
-----------------------------------------------------------------------------------------------------------
12.Sepsis causes a marked metabolic response. Which of the following statements is
false concerning the metabolic response to sepsis?

A=Oxygen consumption is increased in the face of infection.


B=In a patient with a maximal metabolic rate secondary to trauma, the presence of
infection will increase the rate further.
C=Metabolic rate increases at a rate of approximately 10% for each increase of 1°C in
central temperature.
D=The extent of increase in oxygen consumption relates to the severity of the infection.

Answer: B
Oxygen consumption is usually elevated in the infected patient. The extent of this increase
is related to the severity of the infection, with peak elevations reaching 50% to 60% above
normal. If the patient’s metabolic rate is already elevated to a maximal extent because of
severe injury, no further increase will be observed. In patients with only a slightly
accelerated rate of oxygen consumption, the presence of infection will cause a rise in
metabolic rate added to the preexisting state. A portion of the increase in metabolism may
be ascribed to increase in reaction rate associated with fever. Calculations suggest that the
metabolic rate increases 10% to 13% for each elevation of 1°C in central temperature
-----------------------------------------------------------------------------------------
13.The gold standard for evaluation of patients with bile duct strictures is
cholangiography. The two routes for cholangiography are percutaneous transhepatic
cholangiography (PTC) or endoscopic retrograde cholangiography (ERC). Which of the
following statements is FALSE?

A=PTC is generally more valuable than ERC in defining the proximal biliary tree to be
used in reconstruction.
B=PTC is technically easier in patients with bile leaks because the biliary tree is usually
not dilated.
C=Parenteral antibiotics should be administered prior to either procedure to prevent
cholangitis.
D=Biliary stents can be placed via either technique to control biliary leaks.

Answer: B
The gold standard for evaluation of patients with a bile duct stricture is cholangiography.
PTC is generally more valuable than ERC in that it defines the anatomy of the proximal
biliary tree that is used in the surgical reconstruction. ERC is often less useful than PTC
because the discontinuity of the extrahepatic biliary tree usually prevents adequate filling
of the proximal biliary tree. However, in patients with biliary fistulas, the proximal biliary
tree is often not dilated making PTC somewhat technically more challenging. Parenteral
antibiotics should be administered prior to either procedure to decrease the risk of
cholangitis. Biliary stents used either to temporarily control biliary leaks or to stent a
stricture after nonoperative dilatation can be placed by either the percutaneous or
endoscopic route.
14.The management of a suspected bile duct injury depends on a number of factors, most
importantly the mode and timing of presentation. Which of the following statements is
true concerning a patient presenting with a suspected bile leak after laparoscopic
cholecystectomy?

A=Laparotomy should be performed immediately


B=Cholangiography should be performed to determine the nature of the injury
C=Operatively-placed drains should be removed to allow the fistula to close
D=The patient should be discharged to home to allow the leak to close spontaneously
Answer: B
Patients presenting in the early postoperative period with a biliary leak may be septic with
either cholangitis or intra-abdominal bile collections. Sepsis must be controlled first with
broad-spectrum parenteral antibiotics, cholangiography with percutaneous biliary
drainage and percutaneous or operative drainage of biliary leaks. Once sepsis is controlled,
there is no hurry in proceeding with surgical reconstruction of the bile duct stricture. The
combination of proximal biliary decompression and external drainage allows most biliary
fistulas to be controlled or even closed. At that time the external drains may be removed.
The patients can then be discharged to home to allow several months to lapse for resolution
of the inflammation in the periportal region and recovery of overall health status.
-----------------------------------------------------------------------------------------
15.The following statements is true concerning carcinoma of the bile ducts.

A=Carcinoma of the bile ducts is more common than gallbladder carcinoma.


B=Similar to gallbladder cancer, bile duct cancer is more common in females.
C=Unlike gallbladder cancer, there is no association of bile duct cancer and gallstones.
D=Ulcerative colitis is not associated with an increased incidence of bile duct cancer.
E=Choledochal cysts may be associated with the development of bile duct cancer.

Answer: E
Cancer of the bile ducts is even less common than gallbladder carcinoma and is seen in
0.01% to 0.46% of all autopsies. Unlike gallbladder cancer, which is more frequent in
females, males have a higher incidence of bile duct cancer. The average age range of
diagnosis between is 50 and 70 years. Similar to gallbladder cancer, there is an association
between bile duct cancer and gallstones. This association is, however, not as direct as for
gallbladder cancer with only 25% to 57% of patients with bile duct cancer having
gallstones. Biliary tract infection is associated with these tumors; for example, patients
with Clonorchis sinensis infection and chronic typhoid carriers have a higher incidence of
bile duct cancer than the general population. Choledochal cysts are also associated with
bile duct cancer. Patients with ulcerative colitis have a marked increase incidence of bile
duct tumors which tend to follow a more aggressive course.
-----------------------------------------------------------------------------------------
16.Which of the following statements is false concerning the management of
gastroesophageal variceal hemorrhage?

A=Vasopressin decreases portal pressure through the process of splanchnic


vasoconstriction.
B=Somatostatin is as effective as vasopressin but without the cardiac side effects.
C=Balloon tamponade provides short-term control of bleeding esophageal varices.
D=Endoscopic sclerotherapy is more effective than conservative medical therapy in the
treatment of bleeding esophageal varices.
E=Sclerotherapy, although excellent for the control of bleeding short-term, does not
prolong overall survival.

Answer: E
A number of invasive and noninvasive therapies exist for the treatment of bleeding
esophageal varices. Vasopressin is frequently used in the treatment of acute variceal
hemorrhage and acts by decreasing portal venous pressure or flow through splanchnic
vasoconstriction. Vasopressin alone has been reported to temporarily control variceal
hemorrhage in 50% to 75% of patients, however, it can be associated with cardiac and
peripheral extremity cutaneous ischemia. Somatostatin also acts as a vasoconstrictor to
reduce splanchnic flow, with trials demonstrating similar efficacy in controlling acute
hemorrhage when compared to vasopressin but without the cardiac side effects. Balloon
tamponade is generally used for the temporary control of acute variceal hemorrhage
unresponsive to vasopressin or sclerotherapy. Initial control of acute variceal hemorrhage
occurs in about 80% of patients, but bleeding recurs promptly on deflation of the balloons
in over 50%. Endoscopic sclerotherapy has become the primary treatment for bleeding
esophageal varices. A number of clinical trials demonstrate that emergent sclerotherapy is
able to halt variceal bleeding that fails to respond to more conservative measures, with no
increase in frequency or severity of complications. The long-term survival of patients
treated with sclerotherapy continues to be debated. Metaanalysis has been performed on
the data from seven randomized clinical trials evaluating the effect of repeated
sclerotherapy on long-term survival. This analysis demonstrates that sclerotherapy reduces
the number of deaths by 25% therefore supporting the use of sclerotherapy as an effective
means of prolonging survival in patients who have experienced variceal hemorrhage.

: about lactated Ringer`s solution which is true .17

.A- is composed of 130 mEq Na and 130 mEq Cl with a lactate buffer

.B- may induce apoptosis and activate the inflammatory response

C- should not be given to patients with acidosis because of the increase in serum lactic
.acid

.D- should not be used in patients with liver injury

ANSWER B

About lactated Ringer`s solution which is true .18


.A- it is slightly a hypertonic saline

B- has pH of about 6.5

C- it contains a higher concentration of sodium than plasma

D- it is most appropriate for replacement of NGT losses


answer B
L.R à is slightly hypotonic (272 mosmol) ,,, hyposmolar ,,,, provide about 150 ml of free
water with each L given.
Contain Na 130 Cl 109 lactate 28 K 4 Ca 5
Lactate is used rather than bicarbonate because it is more stable in I.V fluids during
storage .
It is converted to bicarbonate in the liver , following infusion , even in the face of
hemorrhagic shock.
Recent evidence has suggested that resuscitation using L.R may be deleterious because it
activates the inflammatory response and induce apoptosis.
Administration of it is appropriate for replacement of small bowel losses and corrections of
ECF deficit .
It can overload in significant situations .
Both L.R and N.S are acidic with respect to the plasma
NS D5 PH =4.5 L.R = 6.5

----------------------------------------------------------------------------------------------------------------
-
:Which is true about normal saline .19
A-is slightly hypotonic
B-may lead to metabolic acidosis
C-contain 154 mEq Na and 130 mEq Cl per L
D-contain 154 mEq Na and 109 mEq Cl per L
E-contain 130 mEq Na and 130 mEq Cl per L

answer B

N.Sà Na 154 (150) , Cl 154(150)


Slightly hypertonic (286)
High chloride content à may load upon the kidneys and may lead to hyperchloremic
metabolic acidosis.
It is an ideal solution for correcting volume deficit associated with hyponatremia ,
hypochloremia and metabolic alkalosis.

------------------------------------------------------------------------------------------------------------
a patient with sodium of 133 , glucose of 240 and blood urea nitrogen of 45 has a . 20
:serum osmolality of
a-225
b-252
c-295
d-305
e-266

answer C

Osmolality =2×Na (mmol) + glucose(mg) + BUN (mg)


--------------- ------------
18 2.8

266+ 240 + 45 = 266+13.3 +16.1 =295


----- ---
2.8 18

:in a normal adult about fluid loss per day which is true .21

.a- about 6000-9000ml through GIT


b-50% of insensible loss as sweating
c-insensible loss is a hypertonic loss
d-a sweating is an active process , and involve loss of hypertonic fluids
e- a sweating is an active process , and involve loss of hypotonic fluids

answer E
Normal person consumes on average of 2L of water per day (75% oral fluid intake and
rest from solid food)
Daily water loss ( 1 L in urine =600-1.5 ,,,,,,,,, 250ml in stool ,,,,,,,,,,600 ml insensible loss)

Insensible losses occur through skin 75% , and respiratory 25%


It is apure water
About 600 ml per day (10 ml/kg/day)
Skin(evaporation not sweating )
Can increase by:
( increase temperature .. one dgree increase above 37.4 lead to increase 250 ml /day in
insensible loss = 15%)
(increase ventilation =tachypnia à 50% increase for each doubling of R.R.)
And hypermetabolism.

Sweating is an active process and involves loss of (hypotonic) electrolyte +water

To clear the products of metabolism the kidneys must excrete a minimum of 500-800 ml of
.urine /day

22.Which of the following statements is true concerning the definitive management of


neck injuries?
A.Patients with evidence of an acute stroke following penetrating injury involving the
carotid artery should be managed with arterial ligation.

B.Unilateral vertebral artery occlusion usually results in a clear neurologic deficit and
therefore revascularization is always indicated.

C.External drainage is an important aspect of the surgical management of an esophageal


injury.

D.Arterial dissection secondary to blunt trauma is best managed by operative exploration


and resection of the dissection.

Answer: c

Blood vessels are the most commonly injured structures in the neck. Major arterial injuries
occur in 18% of penetrating neck wounds and major venous injuries in 26%. Blunt
vascular injures account for a very small percentage of carotid injuries, however their
management is somewhat controversial with treatment highly variable dependent upon the
vascular lesion as well as concomitant injuries. When anatomically feasible,
pseudoaneurysms are probably best managed by resection. The best treatment for arterial
dissection, however, although not completely resolved, would appear to be systemic
anticoagulation if possible to prevent propagation, embolization or thrombosis. Resection
may not be required in the majority of patients. Penetrating carotid injury most commonly
presents with exsanguinating hemorrhage. The indication for repair versus ligation of a
carotid injury depends, in part, on the neurologic presentation. Patients without a
neurologic deficit and a carotid injury should have restoration of vascular continuity with
good neurologic outcome anticipated. Also, patients with all grades of neurological deficits
short of coma should have primary vascular repair. Although experience with
revascularization of patients suffering acute stroke from arteriosclerotic occlusive disease
suggests that hemorrhagic infarction and death may result from revascularization, several
reviews of acute revascularization in the trauma patient note that combined morbidity and
mortality are significantly less in those patients repaired primarily compared to those
managed with arterial ligation. Traumatic injury to the vertebral arteries are now more
commonly identified due to the more liberal application of neck angiography. Unilateral
vertebral artery occlusion seldom results in a neurologic deficit. Treatment of blunt
vertebral artery injury with thrombosis generally is nonoperative: systemic anticoagulation
is recommended to avoid further propagation of existing thrombus.
The diagnosis of esophageal injury can be difficult. The sensitivity of esophography in
detecting esophageal injuries varies from 50% to 90%; the sensitivity of endoscopy ranges
from 29% to 100%. These modalities should be considered complimentary, and when
combined have an accuracy of nearly 100%. Since virtually all reported deaths from
cervical esophageal injuries are the result of delayed or misdiagnosis, a particularly high
index of suspicion is warranted. When injured, the esophagus should be meticulously
debrided and repaired primarily in one or two layers. It is important to drain all such
wounds, because infection or salivary fistula is not an infrequent complication.

23.A middle-aged man is undergoing laparotomy for blunt abdominal trauma. The spleen
and liver are both found to be injured. Which of the following statements is true
concerning the management of these injuries?

A.If the patient has multiple other abdominal injuries and hypotension, splenic salvage
should not be attempted.
B.The incidence of life-threatening sepsis in the adult following splenectomy is no greater
than in the normal population.
C.All liver injuries regardless of their depth require external drainage.
D.The Pringle maneuver should control all bleeding from hepatic parenchymal vessels.
E.If concern for a biliary fistula from the liver parenchyma exists, a T-tube should be
placed even if the common bile duct is otherwise normal.

Answer: A
Solid abdominal organs such as the liver and spleen, are most commonly injured during
blunt abdominal trauma. The management of splenic trauma has been the subject of major
reexamination in the last few decades. Historically, splenic injuries are routinely treated
with splenectomy. With increased appreciation of the danger of post-splenectomy sepsis,
splenic salvage procedures and nonoperative management of these injuries have become
well accepted. This is particularly true in children. The incidence of post-splenectomy
sepsis varies from 0.5% to as much as 12% to 15%, depending on the age and underlying
disease. The incidence is inversely related to age and is higher with underlying
hematologic disorders such as lymphoma or thalassemia. The incidence of life-threatening
sepsis in adult trauma patients is low, but higher than in the normal population. Splenic
salvage should not be attempted if the patient has protracted hypotension or other severe
injuries or if undue delays are encountered in an attempt to repair the spleen.
Simple lacerations of the liver found at the time of surgery do not require drainage unless
they are deep into the liver parenchyma, in which case they have a high probability of
postoperative bile leakage. Biliary fistulas usually will close spontaneously, and major
extrahepatic ductal injuries are rare. A T-tube placed in an otherwise normal common bile
duct is inappropriate unless the extrahepatic biliary tree is injured. In the event that
bleeding continues despite segmental ligation of parenchymal vessels, the structures of the
porta hepatis should be compressed as a diagnostic maneuver (Pringle maneuver). If the
bleeding stops, it is assumed to originate from the portal veins or hepatic artery. If the
bleeding continues, it is presumed to arise principally from the hepatic veins or inferior
vena cava.

24.The management of a patient with frostbite includes:

A.Gradual spontaneous warming


B.Emersion of the tissue in a large water bath with a temperature of 40–42°C
C.Immediate initiation of prophylactic antibiotics
D.Systemic anticoagulation with heparin
E.Immediate debridement of necrotic tissue.

Answer: B
The treatment of frostbite with rewarming should begin in the Emergency Room and not
in the field. Gradual, spontaneous warming is generally inadequate and delayed thawing,
or rubbing the injured part in ice or snow often results in marked tissue loss. Rapid
rewarming should be achieved by immersing the tissue in a large bath of 40–42°C. The
water should feel warm, but not hot to the normal hand. The skin should be gently but
meticulously cleansed, air dried, and affected area elevated to minimize edema. Infection
develops in only about 13% of urban frostbite victims, but half of these infections are
present at the time of admission. Therefore, most clinicians reserve antibiotics for
identified infections. Following rewarming, the treatment goals are to prevent further
injury while awaiting demarcation of the irreversible tissue destruction. The use of
sympathetic blockade, surgical sympathectomy, and intraarterial vasodilating drugs has
generally been ineffective. Heparin, thrombolytic agents, and hyperbaric oxygen have also
failed to demonstrate any substantial treatment benefit. The difficulty in determining the
depth of tissue injury and cold injury has led to a conservative approach to the care of
frostbite injuries. As a general rule, amputation and surgical debridement are delayed for
2–3 months unless infection with sepsis intervenes. The natural history of full thickness
frostbite is gradual demarcation of the injured area with dry gangrene and mummification
clearly delineating a nonviable tissue.
25.Alterations in the immunologic response after a major trauma include ALL
EXCEPT:

A.Decreased CD3 and CD4 population


B.Depression of neutrophil antimicrobial functions including chemotaxis and phagocytosis
C.Decreased levels of pro-inflammatory cytokines including tumor necrosis factor,
interleukin-1, and interleukin-6
D.Impaired macrophage receptor expression and antigen presentation.

Answer: C
Major perturbations in the immune system occur in patients after injury contributing to the
late septic mortality in trauma patients. The changes in the immune system are significant
and global, affecting both humoral and cellular components of the system. Macrophage
receptor expression and subsequent antigen presentation are impaired with similar defects
in lymphocyte function including shifts in T-cell populations with decreased CD3 and CD4
subpopulations, depression of B-cell and immunoglobulin production, and a loss of
antigen recall. Multiple neutrophil antimicrobial functions are suppressed following
trauma including chemotaxis, phagocytosis, respiratory bursts, and intracellular killing.
There are significant changes in humoral mediators following trauma with increased levels
of pro-inflammatory cytokines including tumor necrosis factor, interleukin-1 and
interleukin-6 along with decreased levels of interleukin-2, interleukin-3, and interferon g.

26.Which of the following statements is true concerning penetrating injuries to the


colon and rectum?
A.A patient with 2 or more additional organs injured, significant fecal spillage,
preoperative hypotension, or intraperitoneal hemorrhage exceeding 1 liter should not have
a primary repair of a colon injury.
B.If rectal injury is documented, a loop colostomy provides adequate decompression.
C.Irrigation of the rectal stump should be avoided to prevent contamination via the site of
injury.
D.The rectal wall should be repaired in all cases.
Answer: A
The central issue in the operative management of colonic injuries is the controversy
between primary repair of low-risk colonic injuries and repair or resection with
exteriorization. Primary repair may be selected when additional risk factors have been
excluded. Complications increase with primary repair when there is preoperative
hypotension, intraperitoneal hemorrhage exceeding 1 liter, more than two additional
organs injured, significant fecal spillage, or when more than six hours have elapsed since
injury. Many patients with low-risk penetrating colon injuries can be treated with primary
closure in the absence of these risk factors. High-risk colon injuries are those associated
with severe injuries, as indicated above, and should be treated with resection and
colostomy.
Rectal injuries should be suspected when there is any penetrating injury or a significant
pelvic fracture. Sigmoidoscopic examination is essential. The principles of operative
management include wide debridement of all dead and devitalized tissue and repair of
rectal wall when possible. A totally defunctioning colostomy (not a simple loop colostomy)
is necessary. Retrorectal drainage is indicated only in selected severe injuries. The distal
stump should be washed out to evacuate the fecal contents. Broad-spectrum intravenous
antibiotics, nutritional support, and serial debridements are also indicated.

27.In children who sustain multiple trauma, 25% have serious intraabdominal injuries.
Which of the following statements is true concerning blunt abdominal trauma in
children?
A.Peritoneal lavage plays an important role in the evaluation of the patient.
B.Most pediatric trauma patients will be hemodynamically unstable at the time of
admission.

C.The indications for laparotomy for splenic injury include refractory hypotension or
transfusion requirement in excess of 50% of blood volume within the first 24 hours.
D.Unlike splenic injury, hepatic injury will frequently require exploratory laparotomy.
Answer:C
Diagnostic peritoneal lavage is a rapid and sensitive test for the presence of intraabdominal
hemorrhage. In general, however, peritoneal lavage has a limited role in the care of
pediatric patients as it provides confirmation of a finding, hemoperitoneum, that does not
mandate operation. Unlike adults, 95% of pediatric trauma patients are hemodynamically
stable on admission. The single most useful diagnostic maneuver is a CT scan, which
delineates solid visceral injuries with a high degree of both sensitivity and specificity. The
spleen and liver are injured with about equal frequency in children sustaining blunt
trauma, and together these two target organs account for about 75% of childhood
abdominal injuries.
Experience at virtually every major children’s trauma center in the world supports the
safety and efficacy of nonoperative management of children with splenic ruptures. Most
series report splenic salvage in 90% to 100% of children. Although therapy must be
individualized, the general guidelines are that operation is not indicated until there is
refractory hypotension or a transfusion requirement in excess of 50% of blood volume in
the first 24 hours. In reality, few patients approach this, and transfusion practices are such
that only 10% to 20% of patients with isolated splenic injuries require blood transfusion at
all. The management of liver injuries in pediatric patients has also changed since the
advent of routine abdominal CT scan for blunt trauma. Several reports describe successful
nonoperative treatment of liver injuries detected radiologically in children. This approach
is applicable in most children, with a success rate of 90% and transfusion requirements
similar to those patients with ruptured spleens.
28.The intravenous fluid that a 60 kg., 30-year-old woman with an 80% burn should be
given in the first 24 hours following burn injury is:
A. 19.2 liters of NS.
B. 19.2 liters of lactated Ringer's.
C. 9.6 liters of hypertonic salt solution (sodium concentration 200 mEq. per liter).
D. 7.2 liters of 5% albumin solution.
E. 5.5 liters of the pentafraction component of hydroxyethyl starch.
Answer: B
The consensus range for estimating fluid needs of burn patients in the first 24 hours is 2 to
4 ml. of a physiologic crystalloid solution per kilogram body weight per percent of body
surface burned. In this patient that would be 9.6 to 19.2 liters of lactated Ringer's solution.
The early elevation of circulating levels of catecholamines and glucocorticoids following
burn injury induces glycogenolysis, which results in elevated circulating blood glucose
levels. Glucose should not be administered in the resuscitation fluids, since the resulting
exaggeration of hyperglycemia could induce osmotic diuresis. In the first 24 hours colloid-
containing solution is not commonly used, but if it is used even the Evans formula
recommends only 1 ml. per kg. body weight per percent of body surface burned. Moreover,
patients in one study who received colloid-containing fluids continued to gain weight
during the first 3 postinjury days, retained more sodium, and had less urine output than
patients who received only crystalloid fluids in the first 24 hours. Hypertonic salt solution
is also not commonly used for burn patient resuscitation because of the recently described
and surprisingly frequent occurrence of acute renal failure and increased mortality
associated with its use. If hypertonic salt is used, the amount infused should be less than 9
liters, to avoid excessive elevation of the serum sodium concentration (i.e., above 160 mEq.
per liter). The recommended limit of hydroxyethyl starch infusion is currently 1500 ml. per
day. Although a 10% pentastarch form of hydroxyethyl starch has been used to expand the
plasma volume of burn patients at the end of the first 24 hours, even as little as 500 ml. of
that solution has been reported to prolong both prothrombin and plasma thromboplastin
time.
----------------------------------------------------------------------------------------------------------------
29. Indications for escharotomy of a circumferentially burned right lower limb include
all of the following except:
A. Progressively severe deep tissue pain.
B. Coolness of the unburned skin of the toes of the right foot.
C. A pressure of 40 mm. Hg in the anterior compartment of the distal right leg.
D. Absence of pulsatile flow in the posterior tibial artery.
Answer: B
The blood flow to distal and underlying unburned tissue in a limb can be compromised by
vascular compression due to edema formation beneath the unyielding eschar of a full-
thickness circumferential burn. The most reliably noninvasive means of monitoring
adequacy of the circulation in a circumferentially burned limb is serial examination using
the ultrasonic flowmeter. The absence or progressive diminution of pulsatile flow in the
posterior tibial artery in the lower limb or the palmar arch arteries in the upper limb
indicates a need for escharotomy. Delayed capillary refilling, cyanosis of the digits, and
progressively severe paresthesias, particularly deep tissue pain, are all clinical signs that
may indicate vascular compromise and should be monitored if an ultrasonic flowmeter is
not available. Persistent deep tissue pain and progressively severe paresthesias are the most
reliable of the nonspecific clinical signs. A muscle compartment pressure that exceeds 30
mm. Hg, which is greater than normal capillary pressure, has also been used as an
indication for escharotomy in burn patients. Edema and coolness to the touch of distal
unburned tissue commonly accompany thermal injury and are not useful in assessing the
need for escharotomy.
-----------------------------------------------------------------------------------------------------------
30. ALL of the following are true about inhalation injury in burn patients EXCEPT?
A. A chest x-ray obtained within 24 hours of injury is an accurate means of diagnosis.
B. Its presence characteristically necessitates administration of resuscitation fluids in
excess of estimated volume.
C. When moderate or severe, it exerts a comorbid effect that is related to both extent of
burn and the age of the patient.
D. It increases the prevalence of bronchopneumonia.
E. Prophylactic high-frequency ventilation reduces the occurrence of pneumonia and the
mortality in burn patients with inhalation injury.
Answer: A
Extensive inflammatory changes are evoked in the airway following the inhalation of
smoke and other irritating products of incomplete combustion. Clinical signs are
nonspecific and may be delayed. Chest x-rays are also unreliable in detecting even severe
inhalation injury. Chest x-rays taken within 24 hours of injury were found to be falsely
negative in 92% of 106 patients with inhalation injury. Fiberoptic bronchoscopic
examination is the most reliable single means of diagnosing inhalation injury of the large
airways, but in patients who inhaled finely particulate smoke the large airways may show
little if any inflammatory change. The distance smoke particles travel before deposition in
the airways is inversely related to particle size. When the smoke particle mass median
diameter is less than 0.5 mm., deposition occurs in the terminal bronchioles and alveoli. In
such patients, inhalation injury is best identified by prolonged retention of xenon 133 as
assessed by a ventilation perfusion lung scan.
Historically, fluid restriction was recommended for patients with inhalation injury, but in
recent years it has become obvious that such patients typically require more resuscitation
fluid than the volume estimated by commonly used formulas. Edema of the small airways
and occlusion due to endobronchial sloughing and inspissation predispose burn patients to
develop pneumonia. In one study, 46% of burn patients with inhalation injury developed
pneumonia and 69% of the pneumonias occurred in the first postburn week. The comorbid
effect of moderate to severe inhalation injury is related to both age and burn size and
increases mortality by a maximum of 20% above that predicted on the basis of age and
burn size in patients whose burn injury alone would be associated with a 75% likelihood of
death. In patients with only mild inhalation injury there is little if any increase in mortality
above that predicted on the basis of age and burn size alone. Prophylactic use of high-
frequency percussive ventilation minimizes airway collapse and atelectatic changes, as a
consequence of which the incidence of pneumonia is reduced and survival is significantly
increased.

31.Chylous ascites is the accumulation of chyle within the peritoneal cavity. ALL of the
following statements are true concerning chylous ascites EXCEPT?

A.The cisterna chyli lies at the anterior surface of the first and second lumbar vertebrae
and receives lymphatic fluid from the mesenteric lymphatics.
B.Chylous ascites is most commonly associated with abdominal lymphoma.
C.Paracentesis and analysis of chylous fluid typically reveals elevated triglycerides, protein,
and leukocyte levels, with a predominance of lymphocytes.
D.Treatment of chylous ascites with dietary manipulation will be successful in all cases.
E.The mortality rate in adults with chylous ascites is in excess of 21%.
Answer: D
Chylous ascites is accumulation within the peritoneal cavity of chyle, a lymphatic fluid
with a high lipid content. Access of intestinal lipids to the circulation is via mesenteric
lymphatics that enter the cisterna chyle, which in turn becomes the thoracic duct which
eventually enters the venous system at the junction of the left subclavian and internal
jugular veins. The cisterna chyli lies at the anterior surface of the first and second lumbar
vertebrae slightly to the right of the aorta. Chylous ascites may result from injury to major
lymphatic duct or the cisterna. However for lymphatic leakage to persist, widespread
occlusion of lymphaticovenous collaterals within the abdomen must be present.
Malignancy is the predominant cause (88%) of spontaneous chylous ascites in adults, with
lymphoma the most common malignancy. Diagnostic studies must include not only
documentation of lymphatic origin of the abdominal fluid but also an attempt to delineate
the cause of chylous ascites.
Paracentesis and analysis of chylous fluid typically reveals elevated triglycerides, protein,
and leukocyte levels, with a predominance of lymphocytes. Unfortunately, cytology is
seldom positive despite the presence of malignancy. Lymphangiography may define the site
of lymphatic leak for patients in whom the leak is from the cisterna or retroperitoneal
lymphatics but not when from the mesenteric or hepatic lymphatics. Of noninvasive
studies, CT is the test of choice, with a high diagnostic yield in nontraumatic chylous
ascites in adults. Frequently, laparotomy with node biopsy is required for histology and
typing in cases suspected to be cancer, particularly for lymphoma.
Treatments for chylous ascites have been directed toward decreasing lymph and
triglyceride accumulation. Successful resolution of chylous ascites has been achieved using
a fat-restricted diet with added medium-chain triglycerides in an attempt to reduce
lymphatic transport of triglycerides and perhaps intestinal lymph flow. Although there
have been reports of success using such dietary manipulation, many failures have been
reported. Therefore, in most patients with chylous ascites, treatment is likely to be
successful only when directed toward the underlying cause. For patients with lymphoma,
therapy effective against lymphoma is likely to eliminate chylous ascites.
The prognosis for patients with chylous ascites is much better in infants and children than
in adults, principally because of the differences in causes of the condition. A mortality of
21% is reported in infants and children whereas a mortality of 88% has been noted in
adults. Patients with chylous ascites with associated neoplasms typically have the gravest
prognosis.

32.The following statements is true concerning the epidemiology of inguinal hernias.


A.Inguinal hernias occur with a male-to-female ratio of about 3:1
B.Femoral and umbilical hernias are more common in women, with a female-to-male ratio
of 4:1
C.The frequency of inguinal hernias increases with age
D.Almost all umbilical hernias occur in the pediatric age group.
Answer: C
Inguinal hernias are the most frequently occurring hernia by a factor of five over other
individual types. Umbilical hernias constitute about 14% of hernias, femoral hernias about
5%, and other types are rare. There is a male prevalence in inguinal hernias of about 7:1
(male-to-female), whereas there is a female dominance in femoral and umbilical hernias of
8:1 and 7:1 (female-to-male), respectively. For inguinal hernia, which occurs at all age
levels, frequency increases with age. Umbilical hernias have a bimodal distribution,
peaking in the pediatric population and then in the 40 to 60 year group, in which the
hernias are principally paraumbilical.

33. which of The following statements is true concerning umbilical hernias in adults.

A.Most umbilical hernias in adults are the result of a congenital defect carried into
adulthood
B.A paraumbilical hernia typically occurs in multiparous females
C.The presence of ascites is absolute contraindication to elective umbilical hernia repair.
D.Incarceration is uncommon with umbilical hernias.
Answer: B
An umbilical hernia in a child is usually considered to be congenital. Only about 10% of
umbilical hernias in adults are thought to be the result of a congenital defect carried into
adulthood. Most adult umbilical hernias are acquired and are called paraumbilical
hernias. The paraumbilical hernia typically occurs in a multiparous female. Other patients
with increased intraabdominal pressure, particularly with concomitant chronic abdominal
distension as from ascites, are also at increased risk for the development of paraumbilical
hernias. Umbilical and paraumbilical hernias vary from small to extremely large.
Incarceration is frequent in the large hernias, which typically have a small neck.
Indications for umbilical hernia repair in adults include symptoms, incarceration, large
hernia relative to the neck, and trophic changes in the overlying skin. Among adults with
associated ascites, repair is advocated to avoid potentially serious complications. The
presence of discoloration or ulceration of overlying skin or a rapid increase in size of the
hernia herald impending rupture. Spontaneous rupture of the hernia in these patients can
be catastrophic and is frequently associated with mortality rates approaching 30%. By
comparison, elective umbilical hernia repair can be performed safely in patients with
ascites with acceptable morbidity and mortality.
34.Retroperitoneal fibrosis is a fibrosing condition of retroperitoneum, which is of
significance as it generally encompasses the ureters and eventually causes hydronephrosis
and kidney damage. Which of the following statements is true concerning this
condition?

A.The majority of cases are of a known cause.


B.A history of use of methysergide for treatment of migraine headaches would be
significant
C.There is no known association of malignancy with retroperitoneal fibrosis
D.The disease occurs more commonly in women than in men.
Answer: B
Retroperitoneal fibrosis is a rare condition in which fibrosis develops in the retroperitoneal
space. The ureters frequently will become encompassed by the process eventually causing
hydronephrosis and kidney damage. Retroperitoneal fibrosis occurs most commonly in the
fifth and sixth decades with a 2:1 male-female predominance. The pathophysiology of
retroperitoneal fibrosis remains to be delineated. In fully two-thirds of cases,
retroperitoneal fibrosis is idiopathic, however, an autoimmune process has been suggested
as a potential cause. About 12% of cases of retroperitoneal fibrosis have been associated
with the use of methysergide, a serotonin agonist used for vascular and migraine
headache, and in this subgroup females outnumber males 2:1. Primary or metastatic
malignancy in the retroperitoneum is found in 8% of patients with retroperitoneal fibrosis.
Sarcomas are the most common primary tumors, but non-Hodgkin and Hodgkin
lymphomas and ureteral cancer have also been found. Metastases have originated from
cancer of the stomach, breast, colon, carcinoid, pancreas, prostate, ovary, and cervix. The
focus of tumor may be small but may induce desmoplasia that is grossly indistinguishable
from benign variance of retroperitoneal fibrosis.
------------------------------------------------------------------------------------------------------------
35.which of The following statements is true concerning the anterior abdominal wall
musculature.

A.The lateral musculature of the abdominal wall consists of three muscle layers. These are,
from external to internal, the external oblique, the transversus abdominis, and the internal
oblique muscles.

B.The transversalis fascia lies on the deep side of the transversus muscle and extends to
form an essentially complete fascial envelope of the abdominal cavity.

C.Below the semicircular line, the internal oblique aponeurosis splits into posterior and
anterior laminae.

D.The rectus abdominis muscles originate on the ribs superiorly and on the pubis
inferiorly and are clearly distinct throughout their entire length.
Answer: B
The anterior abdominal wall consists of a group of lateral sheet-like muscles and paired,
longitudinally-oriented flat muscles on either side of the midline. The lateral musculature
of the abdominal wall consists of three layers, each of which has its fascicles running in an
oblique angle to the others. The most superficial of these lateral muscles is the external
oblique muscle. The internal oblique muscle lies deep to the external oblique muscle while
the transversus abdominis muscle is the innermost of the lateral abdominal wall
musculature. The transversalis fascia lies on the deep side of the transversus muscle and
extends to form an essentially complete fascial envelope of the abdominal cavity. The
semicircular line is defined by the lower edge of the posterior sheath about 3 to 6 cm below
the level of the umbilicus, and its convexity is directed superiorly. Above the semicircular
line, the internal oblique aponeurosis splits into posterior and anterior laminae. The
posterior lamina joins with the transversus abdominis aponeurosis to form the posterior
rectus sheath. The anterior lamina fuses with the external oblique aponeurosis to form the
anterior rectus sheath. Below the semicircular line, the internal oblique end transversus
abdominis aponeurosis fuse to form an internal lamina of the anterior sheath, with the
external oblique aponeurosis forming the external lamina of the anterior sheath. The
medial paired rectus abdominis muscles originate on the ribs superiorly and on the pubis
inferiorly. Below the semicircular line, the rectus muscles are nearly fused in the midline
and indistinct, and their posterior surfaces covered only with the transversalis fascia.

36.ALL of the following structures are derived from the external oblique muscle and its
aponeurosis EXCEPT?
A.The inguinal or Poupart’s ligament
B.The lacunar ligament
C.The superficial inguinal ring
D.The conjoined tendon.
Answer: D
The external oblique muscle and its aponeurosis, with its inferiorly and medially-directed
fascicles and the overlying innominate fascia lie deep to the subcutaneous tissue. The
inguinal ligament (Poupart’s ligament) is the inferior edge of the external oblique
aponeurosis and extends from the anterior superior iliac spine to the pubic tubercle,
turning under itself posteriorly and then superiorly to form a shelving edge. Medially, the
inguinal ligament turns under even further to form the lacunar ligament, as part of its
insertion on the pubis. The superficial inguinal ring is a triangular opening in the external
aponeurosis, with its apex superiorly in position slightly above and lateral to the pubic
tubercle, through which the cord exits the inguinal canal. The conjoined tendon is
commonly alluded to in descriptions of inguinal hernia repairs. The conjoined tendon is
the fusion of the aponeurosis of the internal oblique and transversus abdominis muscles.

37. Acute appendicitis is most commonly associated with which of the following signs?
A. Temperature above 104 F(40 C).
B. Frequent loose stools.
C. Anorexia, abdominal pain, and right lower quadrant tenderness.
D. White blood cell count greater than 20,000 per cu. mm.
E. All of the above.
Answer: C
----------------------------------------------------------------------------------------------------------------
-
38.Which of the following most often initiates the development of acute appendicitis?
A. A viral infection.
B. Acute gastroenteritis.
C. Obstruction of the appendiceal lumen.
D. A primary clostridial infection.
E. All of the above.
Answer: C
The majority of patients with acute appendicitis have an obstructed lumen that is due to
either hyperplasia of the lymph follicles in the wall of the appendix or a fecalith. The
obstruction creates a site where the bacteria in the lumen multiply rapidly, producing
exotoxins and endotoxins that then ulcerate the mucosa, allowing pathogenic organisms to
enter the wall of the appendix. An inflammatory process follows that can extend to the
serosa, and penetration through the serosal layer causes generalized peritonitis.
--------------------------------------------------------------------------------------------------------------
39.The diagnosis of acute appendicitis is most difficult to establish in:
A. Persons aged 50 and older.
B. Women aged 18 to 35.
C. Infants younger than 1 year.
D. Pregnant women.
Answer: C
It is very difficult to establish a firm diagnosis of acute appendicitis in an infant of 1 year
or younger since the patient cannot provide a history or be helpful during the physical
examination. It is rare to make a definitive diagnosis preoperatively in such infants, and in
such cases the appendix is usually perforated at the time of operation. While appendicitis is
somewhat more difficult to diagnose in the elderly because of the reduced response to
inflammation; nevertheless, it is usually possible to make the diagnosis. With pregnant
women it is wise to remember that the enlarging uterus in the last trimester dislocates the
appendix higher in the abdomen and that the signs and symptoms follow this anatomic
shift accordingly.
---------------------------------------------------------------------------------------------------------------
40.Once a diagnosis of acute appendicitis has been made and appendectomy decided upon,
which of the following is true?

A. Prophylactic antibiotics should not be administered.


B. Prophylactic antibitics are not necessary unless there is evidence of perforation.
C. If the appendix is not ruptured and not gangrenous, antibiotics may be discontinued
after 24 hours.
D. Multiple antibiotics are in all cases preferable to a single agent.
Answer: C
It is generally held that patients with a diagnosis of acute appendicitis should receive
antibiotics such as cefoxitin or cefotetan. Administration can be discontinued after 24
hours if the appendix is not gangrenous or ruptured. Multiple antibiotics are unnecessary
in straightforward cases.
--------------------------------------------------------------------------------------------------------------
41.The most helpful initial diagnostic radiographic procedure in small bowel obstruction is
A. CT of the abdomen.
B. Contrast study of the intestine.
C. Supine and erect x-rays of the abdomen.
D. Ultrasonography of the abdomen.
Answer: C
----------------------------------------------------------------------------------------------------------
42. The most commonly used imaging method for diagnosis of acute cholecystitis is:
A. CT of the abdomen.
B. Ultrasonography of the gallbladder.
C. Oral cholecystogram.
D. Radionuclide (HIDA) scan of the gallbladder.
Answer: B
---------------------------------------------------------------------------------------------------------------
43. The most diagnostic imaging method of acute cholecystitis is:
A. CT of the abdomen.
B. Ultrasonography of the gallbladder.
C. Oral cholecystogram.
D. Radionuclide (HIDA) scan of the gallbladder
Answer D

--------------------------------------------------------------------------------------------------------------
44.A female patient is seen in the emergency room with reproducible right lower quadrant
tenderness. The approximate incidence of finding a normal appendix on right lower
quadrant exploration in similar nonselected patients is which of the following:
a. 5%
b. 10%
c. 20%
d. 40%
Answer: c
Appendectomy is the most common surgical procedure performed on an emergency basis
in Western medicine. Appendicitis has a negative appendectomy rate of approximately 22%
to 26% in broad based reviews. The perforation rate is as low as 3.6% in a subset of young
males, although this rises substantially when the children or the elderly are included.
Likewise, young females represent a group at particularly high risk for other
intraabdominal pathology.
----------------------------------------------------------------------------------------------------------------
-
45.ALL of the following cause visceral pain from the abdominal organs EXCEPT?
A.Stretching and contraction
B.Traction, compression, torsion
C.Cutting
D.Certain chemicals.
Answer: C
Abdominal pain can be divided into three categories; visceral, somatic, and referred. The
intramural sensory receptors of the abdominal organs are responsible for visceral pain. A
diverse group of destructive stimuli to the abdominal viscera are painless. For example,
almost all abdominal organs are insensitive to pinching, burning, stabbing, cutting, and
electrical and thermal stimulation. The same is true for the application of acid and alkali to
normal mucosa.
The general classes of visceral stimulation that result in abdominal pain include: (1)
stretching and contraction; (2) traction, compression, and torsion; (3) stretch alone; and
(4) certain chemicals. Mediating receptors for these responses are located intramurally in
hollow organs, on serosal structures such as the visceral peritoneum and capsule of solid
organs, within the mesentery and the mucosa. These receptors are polymodal, or
responsive to both mechanical and chemical stimuli. Mucosal receptors respond primarily
to chemical stimulation. Visceral pain almost always heralds intra-abdominal disease but
may not indicate the need for surgical therapy. When visceral pain is superceded by
somatic pain, the need for surgical intervation becomes likely.
46.Prospective studies have shown incidental appendectomy to be advantageous in which
of the following patient groups?

A.Children undergoing staging laparotomy for malignancy who are then palanned for
chemotherapy.
B.HIV infected patients.
C.Patients over 60 years of age.
D.Patients with spinal cord injuries.
E.None of the above.
F.All of the above.
Answer: E
Several studies have looked at incidental appendectomies in a variety of populations. The
deficiency in all past studies of this issue is the lack of prospective long-term trials to assess
the true cost and benefit.
Incidental appendectomy is clearly not indicated in the elderly and in patients undergoing
laparatomy for staging of Hodgkin’s disease. These two specific groups have been shown
to have increased perioperative risks with incidental appendectomy. No prospective studies
have addressed the issue of HIV infected or spinal cord injured patients. While incidental
appendectomies may be performed safely in general, it is difficult to justify any increase in
operative risk without demonstrable benefit.
---------------------------------------------------------------------------------------------------------------
47.Visceral pain is typically:
A.Well localized
B.Sharp
C.Mediated via spinal nerves
D.Perceived to be in the midline.
Answer: D
Peritoneum is a continuous visceral and parietal layer. The nerve supply to each layer is
separate. The visceral layer, i.e., the layer surrounding all intraabdominal organs, is
supplied by autonomic nerves (sympathetic and parasympathetic) and the parietal
peritoneum is supplied by somatic innervation (spinal nerves). The pathways relaying the
sensation of pain differ for each layer and differ in quality as well. Visceral pain is
characteristically dull, crampy, deep, aching and may involve sweating and nausea.
Parietal pain is sharp, severe and persistent. Visceral organs have very little pain sensation,
but stretching of the mesentery and stimulation of the parietal peritoneum cause severe
pain.
Normal embryologic development of the abdominal viscera proceeds with bilateral midline
autonomic innervation that results in visceral pain usually being perceived as arising from
the midline. Epigastric pain is typical of foregut origin. Periumbilical pain signifies pain
emanating from the midgut. Hypogastric or lower abdominal midline pain indicates a
hindgut origin.
----------------------------------------------------------------------------------------------------------------

48.True statement regarding the pathophysiology of acute appendicitis include which


of the following:
A.Fecaliths are responsible for the disease process in approximately 30% of adult patients
B.Lymphoid hyperplasia is a rare cause of appendicitis in young patients
C.Clostridium difficile is implicated as a pathogenic organism
D.Carcinoid tumors account for approximately 5% of all cases of acute appendicitis.
Answer: A
The most common cause of appendicitis is obstruction of the appendiceal lumen. In young
children and young adults, the most common cause of lumenal obstruction is lymphoid
hyperplasia from the submucosal follicles which are abundant. Lymphoid hyperplasia
accounts for 60% of acute appendicitis in the young. In adults, fecalith formation accounts
for approximately 30% of acute appendicitis. There is no known causative relationship of
Clostridium difficile or other specific organisms with acute appendicitis. The normal flora
of the appendix is consistent with that of the adjacent cecum.
Neoplasms of the appendix are rare, occurring in 1% to 1.3% of all appendectomy
specimens. Carcinoid tumors are the most common, followed in frequency by benign and
malignant mucoceles.
----------------------------------------------------------------------------------------------------------
49.A 26-year old woman in her first trimester of pregnancy presents with a 2-day history of
right lower quadrant pain and fever. Physical examination reveals a tender, palpable, right
lower quadrant. There is no evidence of peritonitis or systemic sepsis. Laboratory
evaluation is remarkable for mild leukocytosis, and abdominal ultrasound demonstrates an
inflammatory reactions with suspicious mass but no evidence of abscess. As the surgeon
on call, your recommendation would be.
A.Intravenous hydration, antibiotic prophylasis, and urgent appendectomy
B.Intravenous hydration, antibiotics, bowel rest, and interval appendectomy in 4 to 6 weeks
C.Intravenous hydration, antibiotics, and appendectomy if no improvement in 12 to 24
hours
D.Intravenous hydration, antibiotics, and interval appendectomy when fever has subsided,
leukocyte count has returned to normal, and the patient is pain free
E.Emergent obstetrical consultation for evaluation and treatment of possible ectopic
pregnancy.
Answer: A
The patient presented has a perforated appendix with a phlegmon, but no abscess. One
must routinely provide resuscitation and broad spectrum antiobiotic coverage in this
circumstance. As she is not systemically toxic, it would be rational in a nonpregnant
patient to treat this patient nonoperatively initially and follow this with interval
appendectomy.

However, in this circumstance, the risk of preterm labor associated with anesthesia and
pelvic inflammation increases with more advanced gestation, so the best decision is to
proceed with intravenous hydration, broad spectrum antibiotic coverage and urgent
appendectomy.
------------------------------------------------------------------------------------------------------------
Which of the following statements regarding laparoscopic and open appendectomy is .50
:true
.A.lap.app is associated with a shorter hospital stay by 36 hr
.B.lap.app is associated with decreased postoperative pain
. C.there is no difference in the return time to regular activity
. D.no difference in infection rates exist between the two techniques
.E.lap.app is the procedure of choice in all most all situations
Answer B
:.With lap app
less post op pain -
faster return to work -
.but longer operation time -
.post op stayà rarely reaches a difference of 24 hr -
(.post op infection à wound infection (less with lap -
(à intraabdominal abscess (more with lap

51. Which of the following statements about esophageal anatomy is correct?


A. The esophagus has a poor blood supply, which is segmental in distribution and
accounts for the high incidence of anastomotic leakage.
B. The esophageal serosa consists of a thin layer of fibroareolar tissue.
C. The esophagus has two distinct muscle layers, an outer, longitudinal one and an inner,
circular one, which are striated in the upper third and smooth in the distal two thirds.
D. Injury to the recurrent laryngeal nerve results in vocal cord dysfunction but does not
affect swallowing.
E. The lymphatic drainage of the esophagus is relatively sparse, localized primarily to
adjacent paraesophageal lymph nodes.
Answer: C
Poor technique, not poor blood supply, explains most esophageal anastomotic leaks. While
the major blood supply of the esophagus is from four to six segmental aortic esophageal
arteries, there are extensive submucosal collaterals from the inferior thyroid, intercostal,
bronchial, inferior phrenic, and left gastric arteries. The esophagus lacks serosa and
instead is surrounded by mediastinal connective tissue (adventitia). There are two muscle
layers in the esophagus, an outer longitudinal and an inner circular one. Both layers of the
upper third of the esophagus consist of striated muscle, while in the lower two thirds they
are (nonstriated) smooth muscle. The recurrent laryngeal branches of the vagus nerves
provide both parasympathetic innervation to the cervical esophagus and innervation to the
upper esophageal sphincter (UES). Injury to the recurrent laryngeal nerve therefore
results in improved UES function with secondary aspiration on swallowing as well as vocal
cord dysfunction and hoarseness. The esophagus has extensive lymphatic drainage, with
lymphatic capillaries coursing longitudinally in the esophageal wall and communicating
with paraesophageal, paratracheal and subcarinal, other mediastinal, perigastric, and
internal jugular lymph nodes. This accounts for the biologically aggressive nature of
esophageal carcinoma, which tends to metastasize early in its course.
52.Which of the following statements about gastric polyps is true?
A. Like their colonic counterparts, gastric epithelial polyps are common tumors.
B. They are analogous to colorectal polyps in natural history.
C. Endoscopy can uniformly predict the histology of a polyp based on location and
appearance.
D. Gastric adenomatous polyps greater than 2 cm. in diameter should be excised because
of the risk of malignant transformation.
Answer: D
As early as 1895 Hauser reported an association between familial adenomatous polyposis
of the colon and multiple gastric polyps. This early association may have given rise not
only to the confusing nomenclature of gastric polyps but also to the mistaken notion that
they are analogous to colorectal polyps in microscopic appearance and natural history.
Unlike colonic polyps, gastric epithelial polyps are very uncommon tumors (prevalence
0.4% to 0.8%).
Their histologic appearance cannot be predicted on the basis of location in the stomach,
although the endoscopic literature is beginning to define predictive algorithms based on
location and ultrasound. Multiple polyps are almost always of a single histologic type.
Gastric adenomatous polyps have long been associated with adenocarcinoma. This
association is directly related to the size of the polyps. Up to 24% of polyps 2 cm. or greater
in diameter are associated with adenocarcinoma. In contrast, only 4% of polyps with a
diameter less than 2 cm. are associated with carcinoma. The risk, if any, of carcinoma in
patients with hyperplastic polyps appears to be associated with the atrophic gastritis that
frequently accompanies them rather than with the polyps themselves.
53.Metabolic complications of subtotal gastrectomy with Billroth I or Billroth II
reconstruction include all except:
A. Vit K and Vit B12 deficiency
B. Anemia.
C. Reactive hypoglycemia.
D. Dumping syndrome.
E. Metabolic bone disease.
Answer:A
Anemia develops in as many as 30% of patients within 15 years of surgery. The cause is
multifactorial and includes malabsorption of iron, folate, and vitamin B 12. A metabolic
bone disease occurs in as many as 33% of patients, is similar to osteomalacia, and is
probably a result of malabsorption of calcium and vitamin D. Reactive hypoglycemia
occurs with rapid gastric emptying, resulting in increased glucose absorption immediately
after a meal. Initially there is hyperglycemia, leading to hyperinsulinemia and subsequent
rapid glucose clearance and symptomatic hypoglycemia. Dumping syndrome varies from
very mild symptoms to significantly disabling ones. The severe syndrome occurs in fewer
than 5% of patients. Small, frequent, dry meals of low osmolality reduces symptoms, and
somatostatin analog has been of some clinical use.

54.Which of the following patient scenarios would be best managed with anti-reflux
surgery?
A.A patient with heartburn but normal 24 hour pH monitoring and an intact lower
esophageal sphincter
B.A patient with primarily respiratory manifestations of gastroesophageal reflux, without
typical symptoms of GERD.
C.A patient with increased acid exposure and a mechanically defective sphincter who
responds well to medical therapy but requires continued long-term medication for
continued relief.
D.A patient with gastroesophageal reflux but excessive complaints of epigastric pain,
nausea, vomiting, and loss of appetite.
Answer: c
The first requirement for consideration of anti-reflux surgery is the objective
demonstration of the presence of GERD by 24-hour pH monitoring. Secondly, the patient
must have either symptoms or complications of the disease. Thirdly, the disease should be
caused by defect appropriate to surgical therapy, i.e., a mechanically defective sphincter.
Some patients with increased acid exposure and a mechanically defective sphincter, and
who have no complications of the disease respond well to medical therapy, but they require
long-term medication for continued relief. These patients should be given the option of
surgery as a cost effective alternative.
Atypical symptoms of reflux such as respiratory manifestations often respond well to anti-
reflux surgery. When respiratory symptoms are combined with typical symptoms such as
heartburn and regurgitation, the results of anti-reflux surgery are generally good.
Complaints of epigastric pain, nausea, vomiting, and loss of appetite may be due to
excessive duodenogastric reflux which occurs in about 11% of patients with
gastroesophageal reflux disease. This problem is usually, but not invariably, confined to
patients who have previous upper gastrointestinal surgery. The coexistence of these gastric
symptoms in a patient who also has typical symptoms of GERD should prompt a thorough
evaluation of the stomach using a bile probe, 24 hour pH monitoring or radionucleotide
scanning. In such patients, the correction of only the incompetent cardia can result in a
disgruntled individual who continues to complain of nausea and epigastric pain on eating.
55.Barrett’s esophagus is a complication of gastroesophageal reflux disease. Which of
the following statements IS FALSE concerning this condition?
A.The histologic hallmark is the presence of “specialized” columnar epithelium regardless
of how far it extends into the esophagus
B.Barrett’s epithelium will frequently regress with medical therapy or anti-reflux surgery
C.High grade dysplasia will frequently be associated with foci of invasive carcinoma
D.Patients with adenocarcinoma arising in Barrett’s esophagus have a high incidence of
p53 gene mutations.
Answer: B
Barrett’s esophagus is now recognized as a complication of advanced gastroesophageal
reflux disease. The histologic hallmark of Barrett’s esophagus is the presence of
“specialized” columnar epithelium, which shows features of intestinal metaplasia, easily
recognized by the presence of goblet cells. The presence of specialized epithelium is now
regarded as the pathonomonic feature of Barrett’s esophagus regardless of how high it
extends into the esophagus. Barrett’s esophagus may exist on its own, or may be itself
associated with ulceration, stricture, and malignant change. Once Barrett’s epithelium is
present, medical therapy or anti-reflux surgery rarely causes it to regress. Unless it is
actually ablated, for example with laser therapy, it persists. The most significant feature of
Barrett’s esophagus is its malignant potential. The metaplastic epithelium usually
undergoes dysplastic change prior to becoming frankly neoplastic. High grade dysplagia is
synonymous with carcinoma in situ, and if the esophagus is removed for such a condition,
up to 50% will demonstrate foci of invasive carcinoma.
In the past, the pathophysiology of Barrett’s esophagus was associated with alkaline reflux
on esophageal pH monitoring. However, more recently using a bile sensor for monitoring
bilirubin, this condition is frequently associated with excessive bile in the esophagus.
Repetitive injury from noxious gastric juice can lead during the repair process to mutations
in the p53 gene. Patients with adenocarcinoma arising in Barrett’s esophagus have a high
incidence of p53 mutations.
56.Which of the following statements is correct concerning the diagnostic studies for
esophageal carcinoma?
A.A chest and upper abdominal CT scan is useful for both staging and the best test for
predicting respectability.
B.A barium swallow is an unnecessary test in a patient with dysphagia
C.Bronchoscopy should be performed in all patients with carcinoma of the upper but not
lower two thirds of the esophagus
D.Bone and brain scans should be obtained routinely to rule out distant metastasis
E.Endoscopic ultrasound is a potentially sensitive examination for the staging of
esophageal cancer.
Answer: e
A barium swallow examination is the first study that should be obtained in a patient who
complains of dysphagia. Tumors of the cervical esophagus are difficult to identify by
barium swallow examination and carcinoma of the cardia may be confused with achalasia
or esophageal spasm. Nevertheless, the barium swallow examination localizes obvious
esophageal pathology in preparation for subsequent esophagoscopy and allows the
endoscopist to predict the level at which the tumor is located and the area which requires
the most careful examination. The chest and upper abdominal CT scan is now the standard
radiographic technique for staging esophageal carcinoma. Esophageal wall thickness,
regional adenopathy or pulmonary, liver, adrenal or distant nodal metastasis can be
identified. Although CT is suggested to have a role in evaluating resectability of esophageal
carcinoma, it is particularly limited in its ability to detect invasion of the gastric cardia or
aortic invasion. Bone scan is not warranted unless the patient has specific complaints
suggesting that bone metastases exists. Similarly, routine brain scans are not indicated as
brain metastases from carcinoma of the esophagus are uncommon. Bronchoscopy should
be performed in patients with carcinoma of the upper and middle thirds of the esophagus to
exclude invasion of the posterior membranous trachea or mainstem bronchi, which
precludes a safe esophagectomy. Endoscopic ultrasound is being used with increasing
frequency as an adjunct to the standard radiologic and endoscopic assessment of
esophageal disease. It offers the potential for more sensitive staging of esophageal
carcinoma by detecting the depth of invasion and the presence of abnormal mediastinal
adenopathy.

57.Which of the following statements regarding human gastric acid secretion is


FALSE?
A.Fasting acid secretion, normally 2 to 5 mEq/h, is due to ambient vagal tone and
histamine secretion
B.Truncal vagotomy decreases basal secretion by 80%
C.Histamine2 receptor antagonist administration can decrease basal acid secretion by 80%
D.Fasting acid secretion, normally 5 to 10 mEq/h, is due to circulating levels of gastrin.
Answer: D
Both vagal tone and locally secreted histamine are presumed to be the determinants of
basal acid secretion in humans. Gastrin does not have a role in basal acid secretion in
normal individuals.
Parietal cell activation and the resultant acid secretory response is greater to a combination
of agonists than the sum of the responses to the agents used singly. This increase in
responsiveness is termed potentiation. Potentiating interactions are most apparent when
the stimulants use different second messenger systems, for example, acetylcholine and
histamine. Conversely, blockade of receptors to one stimulant also blocks responsiveness to
the other agonist. Because of this interaction, blockade of histamine receptors by agents
like cimetidine decreases responsiveness to acetylcholine. Blockage of acetylcholine release
by vagotomy decreases responsiveness to histamine secreted by gastric mast cells.
Both vagotomy and histamine2 receptor antagonists decrease basal acid secretion by
approximately 80%.
----------------------------------------------------------------------------------------------------------------
-

58.As a meal is emptied from the stomach, gastric acid secretion gradually returns to
baseline. Which of the following statements incorrectly characterize control
of gastric acid secretion?

A.In humans, the most important inhibitory influence on gastrin release is exposure of the
gastric mucosa to luminal acid.

B.Acidification of the antral lumen causes reciprocal increases in somatostatin release and
decreases in gastrin secretion.

C.Antral distension stimulates gastric acid secretion.


D.Acidification of the duodenal bulb inhibits gastric acid secretion.
E.Exposure of the duodenum to hyperosmolar solutions inhibits acid secretion.
Answer: C
Inhibitory regulation of gastric acid secretion is accomplished by central nervous system,
gastric and small intestinal mechanisms. In humans, the most clearly established gastric
inhibitory influence is suppression of gastrin release by exposure of the antral mucosa to
luminal acid. Antral acidification causes release of gastric mucosal somatostatin which is
linked reciprocally to decreases in gastrin secretion. Antral distension inhibits gastric acid
secretion.
The inhibitory phase of gastric acid secretion begins with entry of the products of digestion
into the proximal duodenum. Acidification of the duodenal bulb and exposure of the
duodenum to hyperosmolar solutions and those containing fat potently inhibit acid
secretion.

59.Which of the following statements is correct with regard to pyloric obstruction


secondary to peptic ulceration?
A.Pyloric obstruction is suggested by hypochloremic hyperkalemic alkalosis.
B.Pyloric obstruction is suggested by hypochloremic hypokalemic alkalosis.
C.Approximately 80% of patients with benign gastric outlet obstruction obtain permanent
relief of symptoms by endoscopically-directed balloon dilatation
D.The lifetime risk of pyloric obstruction in peptic ulcer patients is 40%
Answer: B
Repeated episodes of ulceration and healing can lead to scarring and pyloric stenosis. The
lifetime risk of this complication approximates 10%. Gastric outlet obstruction is
characterized by the development of hypochloremic hypokalemic alkalosis due to loss of
HCl through vomiting and renal compensatory mechanisms that conserve H+ at the
expense of secreted K+. Although 85% of pyloric stenoses are technically amenable to
balloon dilatation, fewer than 1 in 3 will achieve permanent relief of symptoms through
this means.
60.A 50-year-old patient has undergone truncal vagotomy and antrectomy with Billroth II
reconstruction two years ago. The patient now complains of recurrent postprandial pain,
nausea, and vomiting. Endoscopic examination reveals bile in the stomach; endoscopic
biopsies demonstrate histologic evidence of moderately severe gastritis. No other
endoscopic abnormalities are noted. Appropriate therapy include:
a. Octreotide administration
b. Total gastrectomy
c. Conversion of Billroth II gastrojejunostomy to Billroth I gastroduodenostomy
d. Conversion of Billroth II gastrojejunostomy to Roux-en-Y gastrojejunostomy
e. Roux-en-Y hepaticojejunostomy
Answer: d
Symptoms related bile reflux gastritis occur transiently in 10% to 20% of patients after
truncal vagotomy and resection or drainage. Symptoms persist in only 1% to 2%. No
completely satisfactory solution to bile reflux gastritis exists. Medicinal and dietary
treatments have not been proven to be beneficial. Operative diversion of biliary secretions
away from the gastric mucosa by construction of a Roux-en-Y gastrojejunostomy with an
intestinal limb of 50 to 60 cm has been widely reported. The procedure eliminates bilious
vomiting in nearly 100% of patients, but pain persists in up to 30%, and 20% develop
delayed gastric emptying as a result of the procedure.
61.A 50-year-old male with a 2 year history of duodenal ulceration develops sudden, severe
epigastric pain 4 hours prior to evaluation. Physical examination reveals T 38.3, pulse 80,
BP 125/90, diminished bowel sounds, and abdominal muscular rigidity. An upright chest x-
ray reveals pneumoperitoneum. At laparotomy, an anterior perforation in the first portion
of the duodenum is observed. Optimal treatment would include:
a. Omental patch of the perforation followed by truncal vagotomy and antrectomy
after 8 weeks
b. Omental patch of the perforation followed by truncal vagotomy and pyloroplasty
after 8 weeks
c. Omental patch of the perforation followed by chronic cimetidine administration
d. Omental patch of the perforation plus proximal gastric vagotomy
e. Omental patch of the perforation only
Answer: d
Simple omental patching of a perforation in patients with chronic ulcer disease does not
yield satisfactory long-term results. Up to 80% of patients so treated develop recurrent
ulceration and 10% develop secondary complications. A definitive ulcer operation should
be performed during the initial laparotomy if the following circumstances apply: 1) there
has been no preoperative shock, 2) the perforation has been present for less than 48 hours,
and 3) no life-threatening medical co-morbidity exists. Omental patching of the perforation
combined with proximal gastric vagotomy is a preferred approach because it combines
safety, freedom from disabling postoperative symptoms, and a low rate of recurrent
ulceration.

62.With regard to benign gastric ulceration, the most common location of disease is which
of the following?
a. Along the greater curvature
b. Immediately distal to the esophagogastric junction along the lesser curvature
c. In the area of the incisura angularis along the lesser curvature
d. Within the gastric antrum
Answer: c
Gastric ulcers can occur anywhere in the stomach, although they usually present on the
lesser curvature near the incisura angularis. About 60% are located at or slightly above the
angularis. Fifteen percent to 23% of gastric ulcers are within the gastric antrum and 10%
are high on the lesser curvature. Only 5% of gastric ulcers are found on the greater
curvature. In addition, 97% of all gastric ulcers occur within 2 cm of the junctional zone
between fundic and antral mucosa. Gastric ulcers appear at different distances from the
pyloric sphincter because the antrum extends for variable (2 to 16 cm) distances from the
pylorus. It is interesting that with increasing age this junctional zone moves proximally
along the lesser curvature, as does the incidence of gastric ulcer.
63.A 35-year-old smoker is involved in a house fire and receives a 45% total surface area
burn. One half of the burned surface appears to be third degree. On the third post-burn
day, the patient is noted to have bloody drainage from a nasogastric tube and a decrease of
5% in his hematocrit. Appropriate management should include which of the following?
a. Urgent upper gastrointestinal contrast study to delineate site of bleeding
b. Immediate selective arteriography via the left gastric artery to diagnose and treat
presumed stress ulceration
c. Urgent esophagogastroduodenoscopy to diagnosis the cause of bleeding
d. Urgent intravenous infusion of vasopressin at 0.2–0.4 IU/min
Answer: c
Patients who have sustained a major thermal burn of 35% or more of their body surface
area are at a predictably high risk for the development of gastric erosions and hemorrhage.
Endoscopy has demonstrated that gastric erosions are present in 93% of these patients,
whereas the occurrence of severe acute upper gastrointestinal hemorrhage in severely
burned patients ranges between 25% to 50%.
At least 60% of patients at risk develop stress erosions within 1 to 2 days after the
precipitating event. Painless upper gastrointestinal bleeding may be the only clinical sign.
The onset of hemorrhage is often delayed, usually occurring 3 to 10 days after the onset of
the primary disease.
Esophagogastroduodenoscopy is the diagnostic modality of choice to confirm the diagnosis
and to differentiate stress erosion from other sources of upper gastrointestinal
hemorrhage. Correct identification of the bleeding source is made in greater than 90% of
instances. If endoscopy is not diagnostic, visceral angiography through selective
catheterization of the left gastric or splenic vessels may provide information regarding the
primary vessel supplying the bleeding site. In contrast, barium examinations are usually of
little value, due to the superficial nature of stress erosions, and in fact may be detrimental
by interfering with the interpretation of subsequent arteriography.

64.Which of the following statements is correct with regard to gastric bypass for obesity?
a. Rapid weight loss following successful gastric bypass for obesity is associated with
an increased risk of developing cholelithiasis
b. Marginal ulcer develops in 25% of gastric bypass patients
c. Vitamin B12 deficiency is a potential complication of gastric bypass due to gastric
mucosal atrophy
d. Anastamotic leak after gastric bypass is often heralded by bradycardia
Answer: a
The most serious complication after gastric bypass for obesity is anastamotic dehiscence.
Leak is presumed to occur secondary to gastric necrosis due to ischemia from staple line
application or short gastric vessel ligation. Affected patients may have little pain, with
tachycardia, tachypnea and fever as the only manifestations. Physical examination in
morbidly obese patients with peritonitis is unreliable. Marginal ulcers occur in only 10% or
less of gastric bypass patients and respond to H2 receptor antagonists. Rarely,
polyneuropathy has been noted after gastric bypass, usually in association with intractable
vomiting and protein calorie malnutrition. Vitamin B12 deficiency has been noted after
gastric bypass due to decreased acid digestion of vitamin B12 in food; monthly B12
supplementation should be routine. Cholelithiasis occurs in about one third of morbidly
obese patients and gallstone formation is accelerated in the early postoperative period by
the effects of rapid weight loss.

65.Patients with established, complete, simple, distal small bowel obstruction usually have
the following findings on plain and upright abdominal radiographs:
A. Distended small bowel identifiable by the valvulae conniventes.
B. Modest amount of gas in the pelvis.
C. Peripheral, rather than central, distribution of gas.
D. Prominent haustral markings.
E. Free air.
Answer: A
Abdominal x-ray examination of patients with intestinal obstruction usually reveals
abnormally large quantities of gas in the bowel. One can usually identify distended small
intestine or colon. Gas in the small bowel outlines the valvulae conniventes, which usually
occupy the entire transverse diameter of the bowel image. Colonic haustral markings
occupy only a portion of the transverse diameter of the bowel.
Typically, the small bowel pattern occupies the more central portion of the abdomen,
whereas the colon shadow is on the periphery of the abdominal files or in the pelvis.
Patients with mechanical small intestinal obstruction usually have minimal colonic gas, if
any.

66.Which of the following statements about carcinoid syndrome are true?


A. Carcinoid syndrome occurs only when hepatic metastases are present.
B. Serotonin is thought to be responsible for the diarrhea, cardiac lesions, and flushing in
patients with carcinoid syndrome.
C. Foregut carcinoid tumors cause atypical carcinoid syndrome; hindgut tumors are
rarely, if ever, associated with the syndrome.
D. The long-acting vasopressin provides the best symptomatic treatment for carcinoid
syndrome.
Answer: C
Carcinoid syndrome occurs when venous drainage from the tumor gains access to the
systemic circulation, escaping hepatic degradation. Although hepatic metastases are most
often responsible, retroperitoneal metastases and bronchial, ovarian, and testicular
carcinoid tumors can also cause the carcinoid syndrome. Serotonin is thought to be largely
responsible for both the diarrhea and the fibrosing cardiac lesions associated with the
carcinoid syndrome. The vasomotor changes, however, are mediated by kinins and such
vasoactive peptides as substance P, neuropeptide K, neurokinin A, and neurotensin. Other
substances, such as histamine, vasoactive intestinal peptide (VIP), and prostaglandins, may
also contribute to systemic manifestations in the carcinoid syndrome. Foregut carcinoid
tumors, of which stomach and bronchial tumors are the most common, can cause atypical
carcinoid syndrome. It is thought that these tumors are deficient in the enzyme dopa-
decarboxylase and have impaired conversion of 5-hydroxytryptophan (5-HTP) into 5-
hydroxytryptamine (5-HT), leading to secretion of 5-HTP into the vascular compartment.
Some of the 5-HTP is converted into 5-HT and 5-hydroxyindoleacetic acid (5-HIAA) in
extrarenal sites, and some is decarboxylated in the kidney and excreted into the urine as 5-
HT; but some of the 5-HTP is excreted directly into the urine. Thus, in patients with
foregut tumors, the urine contains relatively little 5-HIAA (but more than normal) but
large amounts of 5-HTP and 5-HT, in contrast to patients with midgut carcinoid tumors in
which large amounts of 5-HIAA are secreted into the urine but relatively little 5-HTP.
Carcinoid tumors of the hindgut contain no argentaffin or argyrophil cells, they have no
secretory products, and therefore they are not associated with the carcinoid syndrome. The
long-acting somatostatin analog provides the best symptomatic therapy, because
somatostatin inhibits both release and action of humoral mediators of the carcinoid
syndrome. By contrast, serotonin antagonists are of little value and the efficacy of
.interferon therapy has yet to be established
67.In addition to its absorptive and digestive roles, the small bowel also plays a significant
role in the body’s immune system. Gut-associated lymphoid tissue (GALT) represents a
major division of the immune system. Which of the following statements is true
concerning the immunologic functions of the small intestine?

A.The B lymphocytes of the small intestine do not produce immunoglobulin A (IgA).

B.Peyer’s patches, an example of an aggregated cellular portion of the gut-associated


lymphoid system tissue, are large collections of lymphoid follicles found on the
antimesenteric border of the ileum.

C.The major immunoglobulin of the intestinal immune system is IgM.


D.IgA produced by the intestinal immune system produces the classic Fc-mediated
inflammatory reactions to antigen stimulus.
Answer: b
Gut-associated lymphoid tissue (GALT) represent a major division of the immune system
and is made up of aggregated (Peyer’s patches, lymphoid follicles, mesenteric lymph
nodes) and nonaggregated cellular components. The lamina propria of the small intestine
contains a wide array of nonaggregated lymphoid tissue including B cells, T cells,
macrophages, eosinophils, and mast cells. Some 80% to 99% of B cells are active producers
of immunoglobulin A (IgA). In comparison, only 2% to 5% of B cells found in other
lymphoid tissues of the body secrete IgA. IgA is the major immunoglobulin of the intestinal
immune system. The functional characteristics of IgA are unlike those of other antibodies.
Unlike IgG or IgM, secretory IgA does not induce Fc-mediated inflammatory reactions.
Antigen-IgA complexes do not activate the classic or alternate complement systems, nor
does IgA promote the phagocytosis of bacteria by opsonization. Most of the protective effect
of IgA derives from its ability to bind the threatening antigen efficiently, while resisting
enzymatic degradation by gut enzymes.

68.An 82-year-old female nursing home resident is admitted with massive abdominal
distension and constant abdominal pain with diffuse tenderness. Abdominal x-ray shows a
massively distended loop of colon with a characteristic “bent inner tube” appearance. The
management of this patient should include:

A.Urgent laparotomy because of the massive colon distension, with primary anastomosis.

B.An attempt at endoscopic decompression with a sigmoidoscope ,then Elective


laparotomy and sigmoid resection follow if endoscopic decompression is successful.

C.Urgent laparotomy because of the massive colon distension, with colostomy.

D.If at urgent laparotomy resected bowel is present, colon resection with primary
anastomosis is the best option .

Answer: B
The most common site of volvulus is the sigmoid colon, accounting for 65% of cases. The
preferred method and management involves endoscopic decompression. This conservative
approach resolves the volvulus in 85% to 90% of cases, and elective resection of the
redundant segment can then be planned. Following endoscopic decompression, recurrence
of the volvulus is higher than 60% if sigmoid resection is not performed. If the patient
presents with peritoneal findings, sepsis, and shock, rapid resuscitation followed by urgent
resection and colostomy is warranted.

69. which of The following statements IS TRUE concerning the surgical management
of Crohn’s disease.

A.Strictureplasty, although offering short-term benefits, is associated with a higher rate of


recurrence when compared to resection
B.Frozen section examination of the margin of resection is essential to prevent both
recurrent disease and early anastomotic complications
C.Conservative margins of resection are appropriate, resecting only grossly involved
segments of bowel
D.Patients with Crohn’s disease confined to the colon treated with total proctocolectomy
with construction of an ileal-anal pouch anastomosis.
Answer: C
Surgical therapy for Crohn’s disease is curative not palliative, therefore is reserved for
complications of the disease or failure of or debilitation, secondary to medical therapy. The
lines of bowel resection should be chosen conservatively with only a few centimeters
proximally and distally to the site of visible changes of Crohn’s disease. Microscopic
evidence of Crohn’s disease at the resection margins does not compromise safe
anastomosis and therefore frozen section examination of resection margins is not
necessary. In patients with multiple strictures of the small bowel, resection may involve
excessive resection of bowel. Therefore, strictureplasty is an appropriate surgical therapy.
Long-term results using this approach indicate that recurrence rates are not substantially
increased with strictureplasty, even though inflamed intestinal tissue is left in situ. In
patients with diffuse disease of the colon or rectum, proctocolectomy with ileostomy is the
treatment of choice. Both the risk of ileal involvement and transmural involvement of the
rectum precludes the technique of ileal pouch-anal reconstruction in patients with Crohn’s
disease.

70.Which of the following statements is correct with regard to the use of


carcinoembryonic antigen (CEA) determinations in management of colorectal cancer?

a. CEA determination has 95% specificity when used for screening for colon cancer
development in patients with ulcerative colitis
b. CEA levels are increased in 90% of patients with local recurrence after resectional
therapy
c. CEA measurements are increased in 90% of patients with disseminated disease
d. CEA levels are increased in 90% of patients with local recurrence after resectional
therapy
Answer:C
CEA is a glycoprotein that was originally described to be a tumor-specific antigen derived
from neoplasms of the gastrointestinal tract. CEA is an oncofetal antigen because it is also
expressed by early embryonic or fetal cells. It is now known that CEA is not tumor-specific
since it can be elevated by a variety of malignancies from different sites as well as in some
benign conditions. CEA is not useful as a screening or diagnostic test but is useful as a
tumor marker. CEA is elevated in over 90% of patients with disseminated colorectal cancer
and in about 20% of patients with localized disease. Serum levels generally are elevated in
proportion to the mass of the tumor present and often correlate with response to therapy.
CEA levels are useful when elevated levels fall to normal levels after curative resection. In
about two thirds of patients with recurrent disease, an increased CEA level is the first
indicator of the tumor, and serial CEA testing, combined with regular physical
examinations, is one of the most useful tests for detecting recurrent colorectal cancer.

71.The most common oncogene abnormality observed in association with colorectal cancer
is which of the following?
A. Overexpression of the N-myc oncogene
B. Amplification of the K-ras oncogene
C. Suppression of the erbB oncogene
D.Amplification of the L-myc oncogene.
Answer: B
In colon cancer an important genetic alteration that has been demonstrated is a mutation
of the K-ras protooncogene. The ras protooncogenes are a family of normal genes (N-ras,
H-ras and K-ras) that are highly conserved in nature and encode for the production of
GTP-binding proteins (G proteins) which are important for signal transduction. G proteins
are involved in the transduction of proliferative signals induced by growth factors or
factors involved in cell differentiation. The product of a mutated ras gene is an abnormal G
protein which has lost its ability to become inactivated; and thus, results in continuous
growth stimulation and autonomous cell growth or differentiation. Experimentally,
transfection of normal fibroblasts by mutated ras genes confer neoplastic properties to
those cells.
About half of colorectal carcinomas and a similar percentage of adenomas larger than 1
cm in diameter have been found to have the ras gene mutations. By contrast, less than 10%
of patients with adenomas smaller than 1 cm have this mutation. It has been postulated
that the ras gene mutation may be the initiating event in some colorectal carcinomas or,
alternatively, may promote the clonal expansion of a mutated cell population. It appears
that the ras gene mutation alone is not responsible for tumorigenesis. Other molecular
events appear to be required in addition to ras gene mutations.

72.ALL of the following are tumor suppressor genes that have been associated with the
development of colorectal cancer EXCEPT?
A.The DCC gene
B.The APC gene
C.The P53 gene
D.The Rb gene
Answer: D
Loss of specific chromosomal regions represent genetic alterations that are associated with
the development of colorectal neoplasms in a high percentage of cases. These
chromosomal regions have been hypothesized to contain tumor-suppressor genes whose
products normally regulate growth and differentiation in a negative fashion. One such
gene linked to familial adenomatous polyposis was mapped to the long arm of chromosome
5q and referred to as the APC (adenomatosis polyposis coli) gene. The gene codes for a 300
kilodalton protein which has recently been shown to bind to b-catenin, implying an
important role in cell adhesion and possibly cytoskeleton function. It is hypothesized that
disruption of cell adhesion and cytoskeleton function can lead to loss of contact inhibition
which may promote neoplastic transformation as well as invasiveness of cancer cells.
Another tumor suppressor gene thought to be important in colorectal tumorigenesis is the
p53 gene located in chromosome 17p. Alteration in p53 is one of the most common genetic
events seen in human malignancies. The p53 gene produces a DNA-binding
phosphoprotein that is important in cell proliferation, differentiation and cell survival.
Allelic loss of p53 has been observed in more than 75% of colorectal carcinomas.
Another common genetic alteration associated with colorectal tumors is an allelic loss of
chromosome 18q. This is where the "deleted in colorectal carcinoma" gene, also termed
DCC is located. Mutations in DCC are present in 47% of late adenomas and 73% of
carcinomas. The DCC protein shares significant homology with the neural cell adhesion
molecule family that regulates cell adhesion and recognition.
73.Which of the following types of colonic polyps is associated with the highest incidence
of malignant degeneration?
A.Tubular adenoma
B.Tubulovillous adenoma
C.Villous adenoma
D.Hamartomatous polyp
Answer: c
Adenomas can be divided into tubular (75% to 100% tubular component), tubulovillous
(25% to 75% villous component), or villous (75% to 100% villous) adenomas. The most
common type is tubular adenoma, or adenomatous polyp, which constitutes about 75% of
neoplastic polyps. Tubulovillous adenomas represent 15% and pure villous adenomas 10%
of neoplastic polyps. All adenomas contain some degree of dysplasia or cellular atypia.
This dysplasia can be graded from mild to severe. Carcinoma in situ and severe dysplasia
have been grouped together under the classification of high-grade dysplasia. In carcinoma
in situ, there is no invasion into the muscularis mucosa, as there is in invasive carcinoma.
The incidence of invasive malignancy differs markedly for the three types of adenomas and
increases with size. In general, malignancies are seen in 5% of adenomatous polyps, in
22% of tubulovillous adenomas, and in 40% of villous lesions. Hence, although villous
lesions are much less common, they are more likely to harbor a malignancy.
-----------------------------------------------------------------------------------------------------
74.A 52-year-old man undergoes a right hemicolectomy for a carcinoma of the ascending
colon. Pathological examination of the resected specimen reveals invasion of the tumor to
the level of the muscularis propria. Three of 17 lymph nodes contain microscopic tumor.
What is the correct Dukes classification (Aster-Coller modification) and
associated 5-year survival for this lesion?
A.Dukes C2, 45% 5-year survival
B.Dukes B1, 75% 5-year survival
C.Dukes C1, 45% 5-year survival
D.Dukes B3, 65% 5-year survival
Answer: c
One of the more commonly used staging systems is the modified Astler-Coller system.
According to this system, stage A represents tumors that invade into the mucosa only. Stage
B1 tumors invade into but not through the muscularis propria. Stage B2 lesions invade
through the bowel wall without adjacent organ involvement, whereas stage B3 tumors
involve adjacent organs. Stage C tumors involve regional lymph nodes and are subgrouped
into stages C1, C2, and C3, according to depth of bowel wall penetration. Stage D
represents evidence of distant organ involvement. In general, the 5-year survival rate for
patients with stage D disease is less than 10%. Overall, the 5-year survival rates for stages
A, B, and C are 90%, 77%, and 47%, respectively. Additional studies have revealed that
among Dukes stage C patients, the number of positive nodes is a significant predictor of
survival.
75.An pedunculated polyp, discovered incidentally at colonoscopy, is removed by snare
polypectomy from the ascending colon. Invasive cancer to the level of the submucosa is
identified histologically within the polyp. The lesion is well-differentiated. No lymphatic or
vascular invasion is noted. The cauterized margin is negative for neoplasm.
Appropriate subsequent management includes which of the
following?
A.Repeat endoscopy at 6 months
B.Right hemicolectomy
C.Subtotal colectomy
D.Repeat endoscopy with fulguration of the polypectomy site.
Answer: A
With the availability of colonoscopy, endoscopic polypectomy has become the standard
approach for the treatment of neoplastic polyps. The risk of this procedure is extremely
low, with a complication rate of less than 1%. Almost all pedunculated polyps can be
removed endoscopically with a snare. A dilemma in treating colonic polyps occurs when a
resected lesion contains a malignant focus. A decision must then be made about the need
for a colectomy. If the lesion does not penetrate the muscularis mucosa, it should be
considered an in situ malignancy that does not have the propensity to metastasize and,
therefore, does not require further surgery. If the lesion penetrates the muscularis mucosa,
it is an invasive cancer and may require surgery. In selected cases of pedunculated polyps,
conservative management without colectomy may be undertaken if the lesion does not
contain poorly differentiated tumor cells or evidence of vascular invasion and if a negative
resection margin has been obtained at the level of the stalk. Lesions that are poorly
differentiated or have evidence of vascular invasion, regardless of a negative surgical
margin, should be treated by colectomy.
-----------------------------------------------------------------------------------------------------------
76.Dietary risk factors thought to play a causative role in development of colorectal cancer
include which of the following?
A.High fat intake
B.High fiber intake
C.High smoked food intake
D.High vegetable intake.
Answer: A
Evidence from epidemiological studies suggest that dietary factors play important causative
roles in the development of large bowel cancers. From these studies, fat intake has been the
most consistently positive association and fiber intake the most consistently inverse
association noted in the incidence of colorectal cancer. In comparisons between countries,
the rates of colon cancer are strongly associated with the intake of animal fat and meat.
The associations between per capita consumption of total fat, saturated fat, and cholesterol
and national incidence rates of colon cancer are strongly positive. The proposed
mechanism by which dietary fat may increase the risk of developing colonic cancer is by its
interaction with bile acids.
The relation between fiber intake and colon cancer was initially noted by Burkitt, who
reported low rates of colon cancer in areas of Africa where fiber consumption and stool
bulk were high. In general, epidemiological studies have demonstrated that fiber intake is
higher in non-western countries with lower incidence of colon cancer. The role of fiber
was originally seen simply as providing bulk to dilute potential carcinogens and speed their
transit through the colon. This appears to be an oversimplification and the relation
between fiber intake and colon cancer is more complex. Additional studies have suggested
that certain fibers may bind mutagens, which reduces their contact with colonic
epithelium, may favorably change the fecal pH, and may participate in other complex
interactions.
-------------------------------------------------------------------------------------------------------------
77.Which of the following statements with regard to resection of rectal cancers is true?
A.A distal margin of 5 cm should be obtained because 42% of patients have microscopic
evidence of intramural spread beyond 3 cm from the palpable tumor.

B.A distal margin of 4 cm should be obtained because only 3% of patients have


microscopic evidence of intramural spread beyond 2 cm from the palpable tumor.

C.Local recurrence rates correlate strongly with distal margins less than 4 cm.

D.There is no correlation between local recurrence and free distal margins greater than
2 cm
Answer:D
One of the controversies surrounding sphincter-saving procedures for rectal tumors is the
length of adequate distal mucosal margin. The traditional dictum of 5 cm for a margin is
not substantiated by any studies. Only 2.5% of patients have intramural spread beyond 2
cm from the palpable tumor, and these patients generally have dissemination of tumor
despite aggressive local therapy. There is no correlation between local recurrence and the
extent of distal margin when it is greater than 2 cm. Ideally, a surgical margin of 3 cm,
measured on the fresh specimen, should be achieved.
78.An asymptomatic 52-year-old man is undergoing screening sigmoidoscopy. In the
rectum, at 6 cm from the anal verge, a 2 cm yellow, submucosal nodule is noted. Deep
endoscopic biopsies are consistent with carcinoid. Appropriate management
includes which of the following?
A.Observation
B.Transanal excision
C.Low anterior resection
D.Abdominoperineal resection
Answer: B
Most gastrointestinal tract carcinoids occur in the ileum and the appendix. The rectum is
the next most common site, and occasional carcinoid tumors occur in the colon. Tumor
size is an extremely important prognostic factor. About 60% of rectal carcinoids present as
asymptomatic submucosal nodules measuring less than 2 cm in diameter. Transanal local
excision suffices for definitive therapy since small tumors rarely metastasize. Malignant
potential is seen almost exclusively in patients with tumors larger than 2 cm. More radical
excisions of larger rectal lesions may be required for local control; however, the results of
radical excisions for large rectal carcinoids are poor since they are prone to metastasize.
79.A 72-year-old woman complains of anal itching and burning. Physical examination
reveals an erythematous, scaly lesion, 3 cm in circumference, within the anal canal. The
intersphincteric groove can not be appreciated in the area of the lesion. The remainder of
the physical examination is normal. Appropriate initial management includes
which of the following?
A.Acyclovir 200 mg QID for 10 days
B.Hydrocortisone cream 0.1% topically for 14 days
C.Incisional biopsy
D.Metronidazole 250 mg PO QID for 14 days
Answer: c
Extramammary Paget’s disease may be found in the axilla and in the anogenital region,
including the labia majora, penis, scrotum, groin, pubic area, perineum, perianal region,
thigh, and buttock. Paget’s disease of the perianal area is a malignant neoplasm of the
intraepidermal portion of apocrine glands with or without associated dermal involvement.
Paget’s disease has a long preinvasive phase, but if untreated, an invasive adenocarcinoma
of the apocrine gland type develops. The disease is more common in women than men, with
the highest incidence in the seventh decade.
Macroscopically, the lesion appears as an erythematous scaly or eczematoid plaque-like
lesion, similar to other benign perianal lesions, making clinical diagnosis difficult. A
definite diagnosis is made by biopsy, which shows characteristic histologic appearance—
large, pale, vacuolated cells with hyperchromatic eccentric nuclei. The cells invariably
contain acid mucosubstances, an important feature in distinguishing this lesion from
melanoma and Bowen’s disease.
-------------------------------------------------------------------------------------------------------
80.For the patient in the preceding question, biopsy revealed an invasive apocrine gland
neoplasm. The deep margins included striated muscle infiltrated by neoplastic cells.
Appropriate management includes which of the following?
A.Primary radiation
B.Abdominoperineal resection with bilateral inguinal lymph node dissection
C.Abdominoperineal resection only
D.Carbon dioxide laser fulguration
Answer: c
Wide local excision is the treatment of choice in the absence of invasive carcinoma.
Because of the high incidence of local recurrence and residual tumor, it is vitally important
to obtain an adequate resectional margin. Grossly, the extent of involvement is ill defined,
and multiple punch biopsies may be required to determine the extent of involvement. For
more advanced lesions with underlying carcinoma, an abdominoperineal resection is
indicated. Inguinal lymph node dissection is performed only if groin lymph nodes are
clinically positive for metastasis. Because of the commonly delayed diagnosis (average, 4
years), about 25% of patients with perianal Paget’s disease have metastases when they seek
treatment. The sites of metastases, in order of frequency, are inguinal and pelvic lymph
nodes, liver, bone, lung, brain, bladder, prostate, and adrenal gland. The prognosis is poor
once metastasis has occurred.
------------------------------------------------------------------------------------------------------
81.A 43-year-old woman presents with complaints of anal pain and spotting of blood with
defecation. Physical examination reveals a 2 3 cm area of ulceration within the anal
canal. The remainder of the physical examination is normal. Incisional biopsy is positive
for squamous cell carcinoma. Appropriate management includes which of
the following?
A.Abdominoperineal resection
B.Wide local excision, skin grafting, proximal diverting colostomy
C.Primary radiation therapy
D.Local excision and primary closure.

Answer: c
For localized squamous cell cancers of the anal canal, the most effective protocol consists
of primary irradiation and chemotherapy. The treatment regimen includes:Nigro protocol

1. External irradiation, 3000 rad (30 Gy), to the primary tumor, pelvic, and inguinal
nodes from day 1 to day 21 (200 rad/d, 5 days a week)
2. Systemic chemotherapy, 5-fluorouracil at 1000 mg/m2/24 h, as a continuous
infusion for 4 days, starting on day 1 of radiotherapy and repeated on days 28 through 31
3. Mitomycin C at 15 mg/m2 intravenous bolus on day 1
If the lesion disappears grossly, and its microscopic absence is confirmed by biopsy, no
further treatment is necessary.
82.Recurrent episodes of sigmoid colonic diverticulitis prompt operative therapy. Which of
the following describe the appropriate margins for resection?
A.Proximal margin, splenic flexure; distal margin, rectosigmoid junction
B.Proximal margin, descending colon; distal margin, rectosigmoid junction
C.Proximal margin, descending colon; distal margin, mid-rectum
D.Proximal margin, transverse colon; distal margin, mid-rectum.
Answer: B
At the time of exploratory laparotomy, if the disease is localized, a segmental colectomy
should be performed. The distal extent of the resection should always extend to the
proximal rectum to decrease the chance of recurrence. The proximal extent of resection
should include the segment involved with the acute disease plus any additional colon with
signs of chronic disease or large numbers of diverticula. With this approach, the
recurrence rate after surgical resection is less than 10%. The only absolute
contraindications to primary anastomosis are free perforation with generalized peritonitis;
obstruction with unprepared bowel; and intraoperative conditions that do not warrant
primary anastomosis, such as septic shock, ureteral injury, or other medical conditions that
make a prolonged operation inadvisable. If resection is thought to be unsafe in the
presence of a massive phlegmon or if the patient is too unstable to undergo a resection, a
diverting end colostomy with mucous fistula may be appropriate, with planned colonic
resection at a later date after inflammation subsides.
83.Which of the following statement(s) relating to anal sphincteric function is correct?
A.When the rectum is distended, the internal anal sphincter relaxes and the external anal
sphincter contracts
B.When the rectum is distended, the internal anal sphincter contracts and the external anal
sphincter relaxes
C.The external anal sphincter is responsible for resting anal pressure.
D.The internal anal sphincter is responsible for voluntary continence.
Answer: A
The internal sphincter, because it is innervated by the autonomic nervous system, is not
subject to voluntary control. This powerful muscle exists in a continuously tonic state, and
is responsible for maintaining closure of the resting anal canal. The high-pressure zone of
the anal canal at rest is due to the actions of the internal sphincter. The external sphincter
contributes to anal pressure only when a bolus of stool is present within the anal canal.
The increase in pressure during voluntary contraction (squeeze pressure) is exclusively due
to the activity of the external sphincter. The high resting pressure in the anal canal acts as
a barrier to prevent leakage of mucus and gas.
When the rectum is distended, the internal sphincter relaxes. This relaxation allows the
rectal content to move down to the anal canal. In contrast, when the rectum is distended,
the external sphincter contracts. Reflex contraction of the external sphincter prevents
rectal content from leaking through the anus. Although volitional contraction of the
external sphincter can only be sustained for short periods, it is the most important
mechanism of voluntary continence.
---------------------------------------------------------------------------------------------------------------
84.The most common complication after hemorrhoidectomy is which of the following?
A.Urinary retention
B.Rectal bleeding
C.Incontinence
D.Wound infection
Answer: A
Hemorrhoidectomy should be considered when the hemorrhoids are severely prolapsed
through the anus, requiring manual replacement, or in hemorrhoids complicated with
associated pathology, such as ulceration, fissure, fistula, large hypertrophied anal papilla,
or extensive skin tags. An elliptic excision starts at the perianal skin, includes external and
internal hemorrhoids, and ends at the anorectal ring. The mucosa and submucosa are
dissected off the underlying internal sphincter muscle. Unless there is an associated anal
stenosis or chronic anal fissure, internal sphincterotomy is not performed. The entire
wound is closed with running absorbable suture. The largest and the most redundant
hemorrhoid should be excised first. No packing is placed in the anal canal. Urinary
retention is the most common complication of hemorrhoidectomy, and can be avoided if
intravenous fluids are restricted during the procedure and minimized for the next 6 to 8
hours.
85.The most common location for development of gastrinoma is which of the following?
a. Pancreas to the right of the superior mesenteric vein
b. Pancreatic body and tail
c. Gastric antrum
d. 4 part of duodenum.
TH

Answer: A
The majority of gastrinomas have been identified to the right of the superior mesenteric
vessels within the head of the pancreas or the duodenum. Intraoperative ultrasonography
should be available to assist in tumor localization. In addition, intraoperative upper
endoscopy may be helpful by allowing transillumination of the duodenal wall and
identification of small duodenal gastrinomas. At exploration, any suspicious peripancreatic
lymph nodes are excised and submitted for frozen section. Primary tumors located within
the substance of the pancreas that are small (< 2 cm) and well encapsulated may be
carefully enucleated. Pancreatic tumors without defined capsules or situated deep in the
pancreatic parenchyma may require partial pancreatic resection. In the absence of an
identifiable pancreatic or duodenal tumor, a longitudinal duodenotomy may be performed
at the level of the second portion of the duodenum to allow for eversion of the duodenum in
a search for duodenal microgastrinomas. Primary gastrinomas identified within the
duodenal wall are resected locally, with primary closure of the duodenal defect.
-----------------------------------------------------------------------------------------------------
86.A 35-year-old woman is evaluated for seizure disorder, mental obtundation, and
personality change. Physical examination is normal. Fasting serum glucose is 44 mg/dL.
Other serum values are normal. Subsequent investigations should include
which of the following?
a. Oral glucose tolerance test
b. Determination of postprandial insulin/glucose ratios
c. Assay of serum C-peptide levels, and fasting insulin/glucose ratios.
d. Determination of serum prolactin levels
Answer: c
A common mistake made in the evaluation of a patient with suspected insulinoma is to
commence the evaluation with an oral glucose tolerance test. Instead, insulinoma is most
reliably diagnosed using the technique of a monitored fast. During a monitored fast, blood
for glucose and insulin determinations is sampled every four to six hours, and at the time
of symptom occurrence. Hypoglycemic symptoms typically occur when glucose levels are
less than 50 mg/dl, with concurrent serum insulin levels often being greater than 25 µU/ml.
Additional support for the diagnosis of insulinoma comes from the calculation of the
insulin to glucose ratio (I:G ratio) at different time points during the monitored fast.
Normal individuals will have I:G ratios less than 0.3, while patients with insulinoma
typically demonstrate I:G ratios greater than 0.4 after a prolonged fast. Other measurable
beta cell products synthesized in excess in patients with insulinoma include C peptide and
proinsulin. Elevated levels of C peptide and proinsulin are typically found in the peripheral
blood in patients with insulinoma. The possibility of surreptitious insulin or oral
hypoglycemic agent administration should be considered in all patients with suspected
insulinoma. C peptide and proinsulin levels will not be elevated in patients self-
administering insulin. Additionally, patients self-administering either bovine or porcine
insulin may demonstrate anti-insulin antibodies in circulating blood.

87.Which of the following statements is false concerning the recurrence of breast


cancer?
A.The majority of recurrences recur within five years of diagnosis.
B.More than 70% of breast cancer recurrence involve distant metastases.
C.Pulmonary metastases are the most common initial site of distant recurrence.
D.The local recurrence rate following breast-conserving procedures varies from 10% to
40% whether or not radiation was used
E.Recurrent disease will be seen in at least 20% of node-negative patients undergoing
appropriate primary breast therapy
Answer: c
Metastatic disease following primary therapy for breast cancer can recur at any time.
However, of those who relapse, 50% to 70% do within two years and over 85% relapse
within five years. More than 70% of recurrences are distant, but anywhere from 10% to
30% of recurrences are local. Bone and lung are the most common initial sites of distant
relapse (50% and 25%), respectively. A breast-conserving procedure can be associated with
a local tumor recurrence rate. The rate of local recurrence falls from 40% to 10% if
postoperative radiation therapy is given to the entire breast. Despite potentially curative
resection, at least 20% of node-negative and 60% of node-positive breast cancer patients
have recurrence of their disease at some time after surgery.
88.A 21-year-old woman presents with an asymptomatic breast mass. Which of the
following statements is true concerning her diagnosis and treatment?
A.Mammography will play an important role in diagnosing the lesion
B.Ultrasonography is often useful in the differential diagnosis of this lesion
C.The mass should always be excised
D.The lesion should be considered pre-malignant
Answer: B
Fibroadenoma represents the most common tumor in adolescents and young woman, but if
also frequently encountered in older women. It generally presents as a palpable breast
mass and must be differentiated from cancer. Typically, fibroadenoma presents as a
painless, slow-growing mass found incidentally on breast self examination. Palpation of a
mass usually reveals a well-circumscribed, oval or round, mobile mass with a firm, rubbery
texture. Because the mammographic appearance of a fibroadenoma is rarely
characteristic, mammography plays little role in diagnosing this lesion. Ultrasonography
can differentiate a solid mass from a cyst. Additionally, the ultrasonic appearance of a
well-marginated, homogenous mass may be sufficiently characteristic to permit diagnosis
of fibroadenoma. Excisional biopsy is not necessary for every fibroadenoma. Women under
30 years of age with characteristic physical examination and sonographic appearance of
the fibroadenoma may be given the option of observation. Generally, fibroadenomas are
not felt to be pre-malignant lesions, nor to indicate any increased risk for the development
of breast cancer.
----------------------------------------------------------------------------------------------------------
89.Which of the following are factors associated with an increased risk for developing
breast cancer?
A.Nulliparity
B.Oophorectomy before age 35
C.Menarche at 15 y.
D. Early menopause .
Answer: A
Women who undergo oophorectomy before age 35 and do not take replacement estrogens
have a two-thirds reduction in their breast cancer risk. Replacement estrogen therapy
eliminates the beneficial effect of oophorectomy. Most investigations of oral contraceptive
use do not demonstrate an associated increased risk of breast cancer development. Studies
of estrogen replacement therapy for post-menopausal women have yielded equivocal
results. Most contemporary studies fail to demonstrate an association between breast
cancer risk and post-menopausal use of conjugated estrogens.
BREAST CANCER RISK FACTORS
DEMOGRAPHIC FACTORS
Age more than 30 y
Female gender (130:1 female/male ratio)
GREATLY INCREASED RISK
Known carrier of breast cancer susceptibility gene
Strong family history—two or more first-degree relatives with
bilateral or premenopausal breast cancer
Atypical ductal or lobular hyperplasia or lobular carcinoma in situ
Ductal carcinoma in situ, risk limited to ipsilateral breast
MODERATELY INCREASED RISK
Family history—one or more relatives with breast cancer, not
bilateral or premenopausal
Menstrual history—menarche before age 12 y, menopause after
age 55 y
Parity—nulliparity or first live birth after age 30 y
Radiation—exposure to low-dose ionizing radiation in childhood or
adolescence
Previous breast cancer—low-grade, node-negative, or receptor-
positive; lobular histology
Other cancers—colon or endometrial cancer
Diet—high-fat or high-calorie diet
90.Which of the following statements is correct concerning cystosarcoma phyllodes?
A.The tumor is most commonly seen in post-menopausal women
B.Total mastectomy is necessary for all patients with this diagnosis
C.Axillary lymph node dissection is not necessary for malignant cystosarcoma phyllodes
D.Most patients with the malignant variant of cystosarcoma phyllodes die of metastatic
disease
Answer: c
Cystosarcoma phyllodes is a tumor arising in the mesenchymal tissue of the breast. The
tumors usually present as a painless breast mass. Phyllodes tumor is most commonly
encountered in women age 30–40 years of age but can occur at any age, even before
puberty. The differentiation of a benign from a malignant phyllodes tumor may be difficult.
About one-fourth of all phyllodes tumors are histologically malignant, but only a fraction
of these patients actually develop metastatic disease. The optimum treatment for benign or
malignant phyllodes tumor is wide excision with a margin of normal breast tissue. The
margin must be histologically free of involvement because even benign lesions can recur
after incomplete excision. If this can be done leaving an adequate cosmetic appearance,
mastectomy is not necessary. Total mastectomy is reserved for large lesions in small-
breasted women or recurrences after previous local excision that is not amenable to repeat
local excision. Axillary lymph node dissection is not performed in the absence of biopsy-
proven nodal involvement, even for malignant phyllodes tumors, because axillary
metastases are uncommon.
91.Which of the following statements is true concerning tissue sampling techniques for
breast masses?
A.The sensitivity of fine needle aspiration biopsy is such that mastectomy can be performed
in the case of malignant diagnosis.
B.The accuracy of mammographic-directed fine needle aspiration biopsy is comparable to
that achieved for that of palpable lesions.
C.Core-needle biopsy showing normal breast tissue is an acceptable diagnosis
D.The technique of core-needle biopsy is not applicable to radiographically detected
lesions.
Answer: B
Whatever tissue sampling method is chosen, only biopsy (examination of cells or tissue)
and not physical examination or mammography can establish a definitive diagnosis and
avoid delay in treatment. Fine needle aspiration biopsy (FNAB) permits rapid, minimally
invasive diagnosis of many palpable and some non-palpable, radiologically detected breast
masses. The technique is both reliable and accurate. The incidence of false-positive
findings is generally less than 0.5%. FNAB is not, however, so highly specific that
definitive surgery (particularly mastectomy) should be performed without prior
intraoperative frozen-section confirmation of the presence of cancer. Reported sensitivity
of FNAB ranges from 7% to 99%; with 85% a good estimate of the true sensitivity in
clinically relevant settings. Recently, x-ray-guided FNAB has been used to offer minimally
invasive diagnosis in nonpalpable breast lesions detected mammographically. The
technique is quite effective, especially for mass lesions. Accuracy is comparable to that
achieved with FNAB of palpable lesions. Core-needle biopsy is a helpful tissue sampling
method for palpable masses. The tissue obtained is useful for histologic analysis although
inadequate for cytosol hormone receptor determination. The technique is also applicable
by using mammographic guidance for nonpalpable lesions.
------------------------------------------------------------------------------------------------------
92.A 42-year-old woman undergoes her first mammogram. Clustered microcalcifications
are seen but there is no mass palpable. Which of the following statements is false
concerning this patient’s diagnosis and management?
A.A needle localization and excision of the mass is necessary to establish the diagnosis
B.Frozen-section examination is particularly useful in the diagnosis of this lesion.
C.Intense interlobular fibrosis and proliferation of small ductules with loss of orientation
of lobules and epithelial cells may suggest carcinoma
D.If final diagnosis is Sclerosing adenosis ,ther is no association with an increased risk of
cancer
Answer: B
Sclerosing adenosis is a histologic subtype of fibrocystic change that is not associated with
an increased risk of cancer development. It is, however, one of the benign breast processes
most likely to be confused radiologically and histologically with cancer. Most commonly, it
is detected on routine mammography as cluster microcalcifications without an associated
palpable mass. In these cases, needle localization and excision are required to establish a
diagnosis. Sclerosing adenosis microscopically is characterized by interlobular fibrosis and
proliferation of small ductules. If the fibrous component is particularly intense, the
orientation of lobules and epithelial cells may be lost, mimicking carcinoma.
Differentiating sclerosing adenosis from cancer on frozen-section examination can be
particularly difficult and should not be attempted.

93.Advantages of total thyroidectomy for management of papillary carcinomas of the


thyroid larger than 1.5 cm. include all except:
A. Possibility of using radioactive iodine postoperatively to identify and treat metastases.
B. The ability to use thyroglobulin levels as a marker for recurrence.
C. Lower overall recurrence rate.
D. Lower risk of hypoparathyroidism.
Answer: D
Following total thyroidectomy iodine 131 can be used more efficiently because of the
absence of normal thyroid tissue, which has greater affinity for iodine than papillary
carcinoma tissue. When all normal thyroid tissue is removed, serum thyroglobulin, which
is produced by normal and malignant thyroid tissue, becomes a more effective marker for
recurrence. The overall recurrence rate is lower for patients undergoing total
thyroidectomy, but the risk of hypoparathyroidism is higher for patients who have total
thyroidectomy instead of unilateral lobectomy.
----------------------------------------------------------------------------------------------------------------
-
94.Which of the following statements about follicular carcinoma is false?
A. It presents at a later age than papillary carcinoma.
B. It disseminates via hematogenous routes.
C. It is the most common type of well-differentiated thyroid carcinoma.
D. Extensive angioinvasion portends a poor prognosis.
E. Follicular carcinomas are rarely multicentric.
Answer: c
Follicular carcinoma is more common in older patients (peak incidence in the fifth
decade). The tumor has a marked propensity for vascular invasion and spreads
hematogenously to bone, lung, liver, and central nervous system sites. Local nodal
metastases are less common than in papillary carcinoma. Extensive angioinvasion
indicates a less favorable prognosis. Papillary carcinoma is the most common type of well-
differentiated thyroid carcinomas. Follicular carcinomas are rarely multicentric.
----------------------------------------------------------------------------------------------------------------
-
95. Which of the following statements about the differential diagnosis of hypercalcemia is
correct:
A. Malignant tumors typically cause hypercalcemia by ectopic production of parathyroid
hormone (PTH).
B. The diagnosis of primary hyperparathyroidism is supported by these serum levels:
calcium, 10.8 mg. per dl.; chloride, 104 mmol. per liter; bicarbonate 21 mmol. per liter;
phosphorus, 2.4 mg. per dl.; elevated parathyroid hormone.
C. Familial hypocalciuric hypercalcemia is distinguished from primary
hyperparathyroidism by parathyroid imaging.
D. Although serum albumin binds calcium, the measured total calcium value is usually
unaffected in patients with severe hypoproteinemia.
E. Thiazide diuretics are a good treatment for hypercalcemia and can be given to patients
with apparent hypercalcemia of malignancy.
Answer: B
Malignant tumors rarely secrete PTH itself; they can secrete PTHrP or cytokine activators
of osteoclast activity. The diagnosis of primary hyperparathyroidism is supported by
hypercalcemia with mild hyperchloremic metabolic acidosis and a chloride-phosphate ratio
greater than 33 or a modified chloride (mmol. per liter/mg. per dl.) phosphate ratio greater
than 500. Familial hypocalciuric hypercalcemia is distinguished from primary
hyperparathyroidism by a low urine calcium. Serum calcium changes approximately 0.8
mg. per dl. for every 1 gm. per dl. change in serum albumin. Thiazide diuretics can cause
hypercalcemia and should not be given to patients who are hypercalcemic.
----------------------------------------------------------------------------------------------------------------
-

96.Indications for operation in a patient with previously asymptomatic


hyperparathyroidism include all except:
A. Age older than 60.
B. Nephrolithiasis.
C. A substantial decline in renal function.
D. A substantial decline in bone mass.
E. Depression and fatigue.
Answer: A
Age younger than 50 is considered an indication for operation.
----------------------------------------------------------------------------------------------------------------
-
97.Hyperparathyroidism can affect which of the following organs and body systems?
A. Gastrointestinal tract.
B. Kidneys.
C. Skeleton.
D. Neuromuscular system.
E. Cardiovascular system.
F. all of the above
G. A & E.
Answer F
Gastrointestinal involvement includes pancreatitis and peptic ulcer disease. The kidneys
can be affected by nephrocalcinosis or nephrolithiasis. The bones can be severely affected
by bone resorption. Neuromuscular complaints can include weakness and fatigue.
Hypertension is present in as many as 70% of people with hyperparathyroidism and may be
related to the renal disease.
----------------------------------------------------------------------------------------------------------------
-
98. About Hypoparathyroidism ALL TRUE EXCEPT :
A. Is most commonly encountered as a postviral syndrome.
B. Can be associated with marked hypocalcemia after parathyroidectomy in patients with
bone disease.
C. Can cause anxiety, depression, or confusion.
D. Can cause physical signs such as Chvostek's and Trousseau's signs.
E. Is treatable acutely with intravenous calcium salts and chronically with oral calcium
and vitamin D.
Answer: A
Hypoparathyroidism is most commonly encountered after thyroid surgery. It can be
temporarily severe after parathyroidectomy in patients with bone disease (“postoperative
bone hunger”). The signs and symptoms can include anxiety, depression, confusion,
Chvostek's sign, and Trousseau's sign, as well as circumoral or extremity tingling, tetany
with carpopedal spasms, or seizures. Treatment is as noted.
----------------------------------------------------------------------------------------------------------------
-
99. True statement about pituitary anatomy and physiology include:
A. The pituitary has dual embryonic origin: the anterior pituitary arises from embryonic
ectoderm; the posterior pituitary, from the diencephalon.
B. The hypophyseal portal system integrates function of the anterior and posterior
pituitary.
C. Growth hormone (GH) directly stimulates longitudinal growth of the skeleton and
growth of muscles.
D. Cell types of the anterior pituitary are classified by their position in the anterior
pituitary and by their staining characteristics with histologic dyes.
Answer: A
The anterior pituitary arises from Rathke's pouch (embryonic ectoderm), and it includes
the pars distalis, pars intermedia, and pars tuberalis. The posterior pituitary arises from the
diencephalon and includes the neural stalk, infundibulum, and posterior lobe. The
hypophyseal portal system drains from the hypothalamus and integrates function of the
hypothalamus and anterior pituitary by carrying hypothalamus-derived releasing factors to
target cells in the anterior pituitary. ACTH may be released during stress in response to
interleukin-1, -2, and -6. Somatotrophic actions of GH are indirect and are mediated by the
insulinlike growth factors formerly known as somatomedins. Current classification of
anterior pituitary cells is based on immunochemical identification of their secretory
products (i.e., corticotropes produce ACTH, lactotropes produce prolactin, and thyrotropes
produce thyroid-stimulating hormone [TSH]).
----------------------------------------------------------------------------------------------------------------
-
100. all about Antidiuretic hormone (ADH) are true except:
A. Is related to oxytocin, and both are released from the posterior pituitary in conjunction
with neurophysins.
B. Is released into the circulation by the posterior pituitary in response to a decrease in
plasma osmolality or a decrease in circulating blood volume.
C. May be stimulated by catecholamines and inhibited by phenytoin, alcohol, and lithium.
D. In excess, may produce a syndrome of euvolemic hyponatremia with inappropriately
concentrated urine that is responsive to free water restriction.
E. Deficiency causes prolonged polyuria and polydipsia and may be diagnosed by a
combination of high plasma osmolality and low urine osmolality following water
deprivation.
Answer: B
ADH and oxytocin are nine–amino acid peptides derived from a common ancestral peptide,
vasotocin. ADH is released from the posterior pituitary with neurophysin II in response to
a rise in plasma osmolality greater than 285 mOsm/kg. H 2O. by a 5% or greater decrease
in blood volume, and by catecholamines. The syndrome of inappropriate ADH release
(SIADH) produces euvolemic hyponatremia and is responsive to free water restriction.
Diabetes insipidus (DI) reflects a deficiency of ADH and causes prolonged polyuria and
polydipsia. Diabetes insipidus is diagnosed by a combination of high plasma osmolality and
low urine osmolality following water deprivation. Exogenously administered ADH
differentiates central (ADH-responsive) DI from nephrogenic (ADH-unresponsive) DI.
101.which is true about hypercortisolism :
A. Is most often ACTH-dependent, owing to an ACTH-producing pituitary adenoma.
B. Is best diagnosed by measurement of cortisol from a serum sample collected at 8 A.M.
C. Is attributable to an adrenal source if the basal serum ACTH level is above 10 pg. per
ml., if the hypercortisolism is suppressed by high-dose dexamethasone, and if an adrenal
tumor is visualized radiographically.
D. Commonly caused by small cell carcinoma of the lung, carcinoid tumors, tumors of the
endocrine pancreas, pheochromocytoma, or medullary thyroid carcinoma (MTC).
Answer: A
Cushing's syndrome is most often (80% to 90%) ACTH-dependent and is most often due to
an ACTH-secreting pituitary adenoma (Cushing's disease). Some 10% to 20% of ACTH-
dependent Cushing's syndrome is caused by ectopic production of ACTH from small cell
carcinoma of the lung, carcinoid tumors, tumors of the endocrine pancreas,
pheochromocytoma, and MTC. ACTH-independent Cushing's syndrome secondary to
primary adrenal pathology occurs in 10% to 20% of cases in adults and is the most
common form in children. Measurement of cortisol in two to three consecutive 24-hour
collections of urine is the best screening test for Cushing's syndrome: plasma levels of
cortisol show marked variability, and a single random level is not helpful in establishing
the diagnosis. ACTH-independent Cushing's syndrome suppresses the pituitary and is
diagnosed if the basal serum ACTH level is suppressed below 5 pg. per ml., if the
hypercortisolism is not suppressed by high-dose dexamethasone, or if the metyrapone
stimulation test is negative. The dexamethasone suppression and metyrapone tests do not,
however, distinguish an adrenal from an ectopic ACTH cause of Cushing's since both
suppress the pituitary. Radiographic evidence of an adrenal tumor must be supported by
biochemical testing to confirm that the adrenal is the primary cause of Cushing's
syndrome.
With my best wishes
Dr .Jaber AlFaifi
jaberayf@hotmail.com

Das könnte Ihnen auch gefallen